0% found this document useful (0 votes)
288 views109 pages

Board-Exam-Recall 2017

This document appears to be a list of questions from various topics including biochemistry, anatomy, and other subjects. There are over 100 multiple choice questions listed without any additional context or explanations. The questions cover topics such as amino acid metabolism, protein structure, enzyme kinetics, membrane structure, lipid transport, and anatomical structures like the ureters, uterus, and inguinal triangle among others.
Copyright
© © All Rights Reserved
We take content rights seriously. If you suspect this is your content, claim it here.
Available Formats
Download as XLS, PDF, TXT or read online on Scribd
0% found this document useful (0 votes)
288 views109 pages

Board-Exam-Recall 2017

This document appears to be a list of questions from various topics including biochemistry, anatomy, and other subjects. There are over 100 multiple choice questions listed without any additional context or explanations. The questions cover topics such as amino acid metabolism, protein structure, enzyme kinetics, membrane structure, lipid transport, and anatomical structures like the ureters, uterus, and inguinal triangle among others.
Copyright
© © All Rights Reserved
We take content rights seriously. If you suspect this is your content, claim it here.
Available Formats
Download as XLS, PDF, TXT or read online on Scribd
You are on page 1/ 109

Item #

7
8

10

11

12

13

14

15
16

17

18

19

20

21

22

23
24

25

26

4
5

10

11
12

13

14

15

16

3
4

10

11
12

6
7

10

3
4

10

11
12

13

14

4
5

9
10

11

12

13

14

15

16
17

18

4
5

10

11
12

6
7

4
5

10
11

6
7

10

11

12

1
2

5
6

9
10

11

12

13

14
1

8
9

10

11

12

13

14
QUESTION
BIOCHEMISTRY
The amino acid tryptophan is the precursor of the following EXCEPT:
A. Melanin
B. Melatonin
C. Serotonin
D. Niacin
E. None of the above
The biochemical defect in Type Iia Familial Hypercholesterolemia is:
A. Deficiency in the LDL receptors on the sruface of the cell membrane
B. Absent feedback inhibition of HMG CoA reductase
C. Decrease synthesis of Lipoprotein Lipase
D. Increased conversion of VLDL to LDL
E. None of the above
Which of the following is NOT TRUE regarding protein structure?
A. Denaturation involves hydrolysis of the peptide bonds of the amino acid sequence of a protein.
B. The alpha helix and beta pleated sheet structure of proteins are stabilized by hydrogen bonds.
C. Peptide bonds are considered partial double bonds.
D. Denaturation may be reversible.
E. Peptide bonds are generally in the trans configuration.
Aspartate is metabolized into which of the following by transamination?
A. Alanine
B. Pyruvate
C. Oxaloacetate
D. Lactate
E. Asparagine
The Michaelis constant (Km) is defined as:
A. Maximal number of substrate molecules converted to product per unit time
B. The substrate concentration at which Vi is half the maximal velocity attainable at a particular concentration of enzyme
C. Maximum velocity per mole of enzyme present
D. Maximum velocity divided by the amount of active sites of an enzyme
E. Efficiency in which an enzyme catalyzes a reaction in the forward and reverse directions

The double reciprocal plot of the substrate concentration and velocity refers to the:
A. Michaelis Menten Plot
B. Lineweaver-Burk Plot
C. Eadie-Hofstee Plot
D. Hill Plot
E. Hanes-Woolf Plot
Activity of an enzyme may be regulated by phosphorylation leading to their activation or inactivation. This mode of
regulation is referred to as:
A. Repression
B. Allosteric inhibition
C. Covalent modification
D. Induction
E. Coupling
Carbon monoxide inhibits which complex in the electron transport chain?
A. Complex I
B. Complex II
C. Complex II
D. Complex IV
E. Complex V
Which of the following is TRUE of isoenzymes/isozymes?
A. Enzymes that have identical primary to quaternary structures that catalyze the same reaction
B. Enzymes that catalyze opposing chemical reactions in which one catalyzes the forward and the other catalyzes the
reverse of the reaction
C. Enzymes that have the exact same amino acid sequence but catalyzes different reactions
D. Enzymes that differ in amino acid sequence or are physically distinct but catalyzes the same reaction
E. Hexokinase and glucose-6-phosphatase are considered examples of isoenzymes
Which of the following glucose transporters requires insulin for glucose uptake?
A. GLUT 1
B. GLUT 2
C. GLUT 3
D. GLUT 4
E. GLUT 5
Which of the following coenzymes is dervied from a Vitamin?
A. Coenzyme A
B. Ubiquinone
C. Tetrahydrobiopterin
D. Lipoic acid
E. None of the above
A medical student ate food that was expired hence suffered food poisoning with severe diarrhea for 7 days. A few days after
recovering, he would still have difficulty digesting which carbohydrate?
A. Dextrins
B. Maltose
C. Sucrose
D. Isomaltose
E. Lactose
A 12 year old male presents with fatigue, polydypsia, polyuria and polyphagia. He is diagnosed with Type I DM. Which of
the following is most likely occuring in the patient?
A. Increased fatty acid synthesis from glucose in the liver
B. Decreased conversion of fatty acids to ketone bodies
C. Increased stores of triacylglycerol in adipose tissue
D. Increased production of acetone
E. All of the above
55 year old male alcoholic was brought to the emergency room in an unconscious state. The attending doctors administered
intravenous glucose and thiamine which greatly improved the patient's status after several hours. Which enzyme of
carbohydrate metabloism is dependent on thiamine?
A. Glucokinase
B. Phosphofructokinase 1
C. Pyruvate kinase
D. Phosphoglycerate kinase
E. Pyruvate dehydrogenase
What form/s of Vitamin A is required for the replenishment of rhodopsin for vision?
A. Retinoic acid
B. All trans retinal
C. 11-cis retinal
D. Beta cryptoxanthin
E. Beta carotene
Decarboxylation of the ketone acetoacetate yields which compound?
A. Acetone
B. B-hydroxybutyric acid
C. Hydroxymethylglutaryl CoA
D. Alanine
E. Aspartate
A 25 year old female has been fasting completely for 10 days, drinking only water. Synthesis of ketone bodies is stimulated
to serve as alternative fuel for peripheral tissues. Which organ in the body CANNOT utilize ketone bodies as a source of
energy?
A. Skeletal muscle
B. Liver
C. Cardiac muscle
D. Kidney
E. Brain
In nucleosides, the ribose is linked to the N-1 or N-9 of a pyrimidine or purine base respectively via a/an:
A. Ether bond
B. Ester bond
C. Glycosidic bond
D. Peptide bond
E. Amide bond
Medium chain fatty acids are given during fat malabsorption conditions because they:
A. Provide more fuel than long-chain fatty acids
B. Enter directly into the portal blood and can be metabolized by the liver
C. Are activators of lipoprotein lipase
D. Are more efficiently packed into serum lipoproteins
E. None of the above
Which of the following correctly describes biological membranes and their properties?
A. The major lipid found in membranes are phospholipids
B. Integral proteins are found on either sides of the lipid bilayer
C. The fluidity of membranes with abundant unsaturated fatty acids is increased by cholesterol
D. Phospholipids in the membranes are not capable of lateral motion
E. All of the above
In fasting and starvation, the activity of all of the following enzymes are expected to increase EXCEPT:
A. Glycogen phosphorylase
B. Glycogen-6 phosphatase
C. Pyruvate kinase
D. Pyruvate carboxylase
E. HMG CoA synthase
In the calculation of total energy requirement, importance is drawn most on this component because it presents the greatest
variability among individuals:
A. Basal metabolic rate
B. Desirable body weight
C. Physical activity
D. Thermogenic effect of food
E. Total body surface area
This is an automated method that exponentially amplifies targeted DNA sequence in vitro by repeated cycles of enzymatic
DNA: synthesis
A. Chromosome walking
B. Overlap hybridization
C. Polymerase chain reaction
D. Restriction fragment length polymorphism
E. Southern blot
Brain and nerve tissues, including myelin sheath, particularly contain these lipids:
A. Fatty acids
B. Gangliosides
C. Sterols
D. Waxes
E. Phosphatides
15 year old female was brought to the ER because of severe abdominal pain. Blood was taken and was found to be milky in
appearance. The triacylglycerol level is in excess of 2,000 mg/dL. The patient was placed on a diet severely limited in fat,
but supplemented with medium-chain fatty acids. Which of the ff lipoprotein particles are most likely responsible for the
appearance of the patient's plasma?
A. Chylomicrons
B. VLDL
C. LDL
D. IDL
E. HDL
40 year old female presents with an LDL serum level of 400mg/dL and a triacylglycerol level of 170 mg/dL. She is
diagnosed with Type II familial hypercholesterolemia. In this disorder, a mutated LDL receptor is formed, such that it cannot
bind to LDL. Which of the following would result?
A. Cellular HMG CoA reductase activity is not inhibited
B. The triacylglycerides in chylomicrons cannot be degraded
C. The VLDL level in the serum increases
D. The HDL level in the serum increases
E. All of the above

ANATOMY
Which of the following is NOT TRUE of the ureter?
A. The ureters have 3 anatomic constrictions
B. The ureters lie posterior and medial to the ovarian vessels within the suspensory ligament of the ovary
C. The ureters lie inferior and posterior to the uterine arteries
D. The ureters enter superolaterally at the base of the urinary bladder
E. None of the above
All of the following supply blood to the ureters EXCEPT:
A. Superior vesical artery
B. Inferior vesical artery
C. Renal artery
D. Gonadal artery
E. None of the above
Pelvic examination done on a patient shows that the uterine body is tipped away from the urinary bladder and the fundus is
tipped posteriorly. Which of the following describes the position of the uterus?
A. Anteverted, Anteflexed
B. Anteverted, Retroflexed
C. Retroverted, Anteflexed
D. Retroverted, Retroflexed
E. Retropulsed, Retroverted
Which of the following is NOT TRUE of the Hesselbach's triangle?
A. Medial boundary is formed by the lateral border of the rectus abdominis
B. Poupart's ligament forms the inferior and lateral boundary
C. Inferior epigastric artery forms the superior and lateral boundary
D. Hernia located within this triangle is termed an indirect inguinal hernia
E. None of the above
A patient was involved in a car accident and dislocated his left hip, damaging a nerve and had difficulty extending his left
thigh over the hip joint. Which of the following nerves most likely was affected?
A. Femoral nerve
B. Obturator nerve
C. Tibial division of sciatic nerve
D. Common peroneal division of sciatic nerve
E. Saphenous nerve

Which of the following bones forms part of the floor of the orbit?
A. Frontal
B. Sphenoid
C. Ethmoid
D. Palatine
E. Lacrimal
What stage of a neutrophil precedes the band form?
A. Myeloblast
B. Promyelocyte
C. Myelocyte
D. Metamyelocyte
E. Normoblast
An 8 month old baby boy was brought to the emergency room because of unresponsiveness and seizures. The caregiver
admitted that she shook the baby boy vigorously out of shear frustration from his incessant crying and noticed thereafter
that the child was becoming lethargic and had sudden onset of generalized seizures . An impression of shaken baby
syndrome was made. The child most likely suffered from a/an?
A. Subdural hematoma
B. Epidural hematoma
C. Subgaleal hemorrhage
D. Lacunar infarct
E. Intracerebral hemorrhage
Which of the following structures is NOT contained in the carotid sheath
A. Internal jugular vein
B. Vagus nerve
C. Recurrent laryngeal nerve
D. Common carotid artery
E. Internal carotid artery
Which of the following is TRUE regarding the gag reflex?
A. The afferent limb is supplied by the trigeminal nerve
B. The efferent limb is supplied by the glossopharyngeal nerve
C. The afferent limb is supplied by the facial nerve
D. The efferent limb is supplied by the vagus nerve
E. The efferent limb is supplied by the hypoglossal nerve
The spring ligament of the foot, helps in maintaining the medial longitudinal arch of the foot. Which of the following correctly
describes the ligament?
A. It is a strong ligament and is the main bond of union between the talus and calcaneus.
B. It is a strong band of ligament that runs from the anterior margin of the sustentaculum tali of the calcaneus to the
inferior surface and tuberosity of the navicular bone
C. It is a Y-shaped ligament whose stem is attached to the calcaneus and its lateral and medial limbs attached to the
cuboid and navicular bones respectively.
D. It is strong ligament attached to the undersurface of the calcaneus and is connected to the cuboid, the bases of the
third, fourth, and fifth metatarsal bones.
E. It is a wide strong ligament attached to the anterior tubercle of the undersurface of the calcaneus to the adjoining
cuboid.
Which of the following veins drain directly into the right atrium?
A. Anterior cardiac vein
B. Great cardiac vein
C. Small cardiac vein
D. Middle cardiac vein
E. All of the above
Which of the following structures can be found in the anterior mediastinum?
A. Vagus nerve
B. Azygous vein
C. Hemiazygous vein
D. Thymus
E. Esophagus

All of the following structures can be found in the deep perineal space EXCEPT:
A. Cowper's gland
B. Sphincter urethrae muscle
C. Dorsal nerve of the penis/clitoris
D. Internal pudendal vessels
E.
A. Root of the
It is also penis/clitoris
called the neurohypophysis or pars nervosa.
B. It is derived from the downgrowth of nervous tissue from the hypothalamus to which it remains joined by the pituitary
stalk.
C. It is largely composed of non-myelinated axons whose cell bodies are located in the supraoptic and paraventricular
nuclei.
D. The axons are supported by specialized highly branched glial cells called pituicytes.
E. None of the above

Which of the following muscles depresses the mandible/jaw?


A. Medial pterygoid
B. Lateral pterygoid
C. Masseter
D. Temporalis
E. Depressor labii inferioris

MICROBIOLOGY
Vancomycin resistance in Staphylococcus aureus is carried in which gene?
A. mecA
B. vanA
C. blaZ
D. dfrB
E. gyrA
What is the preferred way of isolating mumps virus?
A. Embryonated hen's egg
B. Monkey kidney cells
C. Mice fibroblasts
D. Mice lung cells
E. None, there is no way of isolating mumps virus
Respiratory syncytial virus infection in the young usually leads to bronchiolitis or pneumonia. What is the usual
symptomatology when a healthy adult is reinfected with the said virus?
A. Usually limited to an upper respiratory tract infection, resembling a cold
B. Often leads to moderate to severe pneumonia requiring hospitalization
C. Adults may develop recurrent paroxysmal attacks of wheezing, similar to asthma
D. A great majority of elderly adults who are reinfected develop significant morbidity and mortality
E. Reinfection in adults is rare
Which of the following is NOT TRUE regarding herpes simplex virus (HSV) infection?
A. Viremia is more common during primary HSV-2 infections than during HSV-1 infections.
B. Primary HSV infections are usually mild, in fact most are asymptomatic.
C. Viral persistence in latently infected ganglia lasts for the lifetime of the host.
D. Spontaneous reactivations occur despite HSV-specific humoral and cellular immunity in the host.
E. Recurrent infections are more extensive and severe.
24 year old sexually promiscuous male comes in due to 1 week history of papules, vesicles and ulcers over his penile shaft
that resolved spontaneously over a few days. Afterwhich, he developed painful inguinal lymphadenopathy, with the
overlying skin over his groin area gradually turning purplish, eventually suppurating forming sinus tracts. What is the most
likely causative organism?
A. Treponema pallidum
B. Haemophilus decreyi
C. Herpes simplex virus type 2
D. Klebsiella granulomatis
E. Chlamydia trachomatis
In relation to the above case, you decided to treat him with the appropriate antibiotics. What is a good way of documeting
eradication of the organism?
A. Repeat smears of the pus or buboes showing a decrease in the number of organisms
B. A drop in the complement-fixation (CF) titers
C. Immunofluorescence
D. Repeat culture using McCoy cell cultures
E. Both C & D
Persistence of this hepatitis profile marker denotes chronic hepatitis B infection:
A. Anti-HBs
B. Anti-HBc IgM
C. Anti-HBs IgM and Anti-HBc IgM
D. Anti-HBc IgM and Anti-HBe
E. HBsAg
Gram stain of Bacillus anthracis would show:
A. Gram positive non-motile rods
B. Gram positive cocci in clusters
C. Gram positive cocci in pairs or chains
D. Gram negative diplococci
E. Gram negative motile rods
Most common mode of transmission of anthrax is through:
A. Inhalation
B. Direct contact with infected animals
C. Body fluids
D. Ingestion of spores
Yersinia pestis, the organism responsible for the historic bubonic or black plague is transmitted via:
A. Rodent bite
B. Flea bite
C. Mosquito bite
D. Rodent urine
E. Improperly cooked meat
Which "atypical" mycobacteria produce human disease indistinguishable from tuberculosis?
A. Mycobacterium fortuitum
B. Mycobacterium avium complex
C. Mycobacterium kansasii
D. Mycobacterium leprae
E. Mycobacterium marinum
Which of the following is NOT TRUE regarding smallpox?
A. Smallpox is caused by a brick-shaped, complex, linear dsDNA virus.
B. Smallpox characteristically have lesions in the same stage of development in contrast to chickenpox that have different
stages of skin lesions
C. Portal of entry is through the mucus membranes of the upper respiratory tract.
D. One of the reasons regarding the success of its eradication is that there is only a single serotype of the virus.
E. None of the above

PHYSIOLOGY
A pharmaceutical group developed a drug that can inhibit phospholamban. What would be its effect on cardiac myocytes?
A. There would be cardiac muscle relaxation
B. SERCA would be uninhibited and there would be more uptake of calcium ions into the sarcoplasmic reticulum
C. It would produce a positive inotropic effect
D. It would cause a decrease in activation of the calcium dependent phosphatases implicated in cardiac hypertrophy
E. All of the above

Patient suffered a stroke and developed apraxia. Which area/s of the brain when damaged could potentially cause apraxia?
A. Posterior parietal lobe
B. Premotor cortex
C. Primary motor cortex
D. A & B
E. B & C
The representational hemisphere:
A. Is the right cerebral hemisphere in most right-handed individuals
B. Is the left cerebral hemisphere in most left-handed individuals
C. Includes the part of the brain concerned with language functions
D. Is the site of lesions in most patients with aphasia
E. Is morphologically identical to the opposite nonrepresentational hemisphere
Lymphocytes:
A. All originate from the bone marrow after birth
B. Are unaffected by hormones
C. Convert to macrophages in response to antigens
D. Are part of the innate immune response
E. Are part of the body's defense against cancer
Red blood cells are considered powerful acid-base buffers. When carbonic acid dissociates into hydrogen and bicarbonate
ions, most of the hydrogen ions bind to hemoglobin and the bicarbonate ions diffuse out of the red blood cells into the
plasma in exchange for what ion?
A. Sodium
B. Phosphate
C. Potassium
D. Chloride
E. Magnesium
Gastric pressures seldom rise above the levels that breach the lower esophageal sphincter, even when the stomach is filled
with a meal, due to which of the following processes?
A. Peristalsis
B. Gastroileal reflex
C. Segmentation
D. Stimulation of the vomiting center
E. Receptive relaxation
The pressure in a capillary in skeletal muscle is 35 mmHg at the arteriolar end and 14 mmHg at the venular end. The
insterstitial pressure is 0 mmHg. The colloid osmotic pressure is 25 mmHg in the capillary and 1 mmHg in the interstitium.
The net force producing fluid movement across the capillary wall at its arteriolar end is:
A. 3 mmHg out of the capillary
B. 3 mmHg into the capillary
C. 10 mmHg out of the capillary
D. 11 mmHg out of the capillary
E. 11 mmHg into the capillary
Which of the following hormones acts on its target tissue/s using inositol 1,4,5-triphosphate (IP3) as a second messenger?
A. Insulin
B. Atrial natriuretic peptide
C. Cortisol
D. Parathyroid hormone
E. Thyroid releasing hormone

Which of the following would cause a shift to the right in the oxygen-hemoglobin dissociation curve?
A. Increase in pH
B. Decrease in CO2
C. Decrease in temperature
D. Exercise
E. Decrease in 2,3 bisphosphoglycerate
In third-degree or complete heart block:
A. Fainting may occur because the atria are unable to pump blood into the ventricles
B. Ventricular fibrillation is common
C. The atrial rate is lower than the ventricular rate
D. Fainting may occur because of prolonged periods during which the ventricles fail to contract
E. All of the above

LEGAL MED
How many Continuing Professional Development (CPD) units is required for the renewal of professional identification cards
for physicians in the Philippines?
A. 15 credit units
B. 30 credit units
C. 45 credit units
D. 60 credit units
E. 120 credit units

What test is being described where a ligature is applied around the base of the finger with moderate tightness to check if
there would be change in color as a means of identifying whether a person is alive or dead?
A. Magnus test
B. Icard's test
C. Diaphanous test
D. Winslow test
E. Walker test
A botched rhinoplasty was done by a plastic surgeon on a 25 year old female. Because of this, the patient now has fear of
going out into public places because of humiliation. Which of the following damages can be awarded to this patient?
A. Moral damages
B. Corrective damages
C. Nominal damages
D. Moderate damages
E. Liquidated damages
Which type of fee is measured by the space of time and not by the quality or quantity of medical services offered?
A. Contingent fee
B. Retainer fee
C. Simple contractual fee
D. Dichotomous fee
E. Straight Fee
Which part of a firearm functions to transform mechanical energy by the hit of the firing pin on the percussion cap to
chemical energy via combustion?
A. Cartridge
B. Primer
C. Propellant
D. Projectile
E. Muzzle
The art and study of recording fingerprints as a means of identification is:
A. Dactylography
B. Dactyloscopy
C. Poroscopy
D. Bibliotics
E. Graphology
This refers to a woman having an intact hymen but the orifice is wide and elastic to admit 2 or more fingers:
A. Moral virginity
B. Virgo intacta
C. Demi-virginity
D. Flase physical virgin
E. True physical virgin
An administrative complaint was filed with the Professional Regulatory Commission against a physician who was eventually
found guilty. This may result in:
A. Reprimand, suspencion, or revocation of license to practice
B. Imprisonment for criminal negligence
C. Civil damages paid to the patient to compensate for injuries
D. Need to be recertified
E. All of the above
After how long can a doctor apply for reinstatement of his license after revocation?
A. 12 months
B. 18 months
C. 24 months
D. 36 months
E. Cannot apply for reinstatement anymore once revoked
While driving in EDSA, Dr. X witnessed the shooting of a politician by a group of armed assailants. He was subpoenaed to
appear in court because of this. He relented nevertheless and appeared as a/an:
A. Expert witness
B. Ordinary witness
C. Chanced witness
D. A & B
E. All of the above
If the doctor in the above case did not attend to a subpeona citing as resaon that he has a lot of out-patient department
patients to see, he may be cited for:
A. Direct contempt of court
B. Indirect contempt of court
C. Disrespect for the court
D. Unduly prioritizing his clinic practice
E. Criminal Negligence
A medical witness need not comply with a subpeona when:
A. The court issuing the subpeona has no jurisdiction over the subject matter and person of the case
B. The place of residence is >100km from the court issuing the subpoena
C. The medical doctor is attending to an emergency and no one is available and competent enough to be his substitute
D. The physician is suffering from an infirmity which may be aggravated if ever he complies
E. All of the above
This test is utilized to determine whether semen is of human origin or not:
A. Florence test
B. Berberio test
C. Puramen reaction
D. Acid phosphatase test
E. Biological test of Farnum
If a person died of a communicable disease, the body must be buried within:
A. 12 hours
B. 24 hours
C. 36 hours
D. 48 hours
E. 72 hours

PATHOLOGY

Which of the following does NOT describe apoptosis?


A. Pyknosis
B. Plasma membrane remains intact
C. No inflammatory response is incited
D. May be pathologic
E. None of the above
Eosinophils granules contain this substance, a highly charged cationic protein that is toxic to parasites but also causes
epithelial cell necrosis:
A. Eotoxin
B. Lysozyme
C. Defensins
D. Major basic protein
E. LTC4
40 year old female consulted you because of blurring of vision and foreign body (sandy) sensation under both eyelids with
associated dysphagia and dryness of the mouth. What pathologic mechanism/s could explain her condition?
A. CD4+ T cell reactions against unknown antigens in the ductal epithelials cells of the exocrine glands
B. Systemic B cell hyperactivity as evidenced by the presence of ANAs and rheumatoid factor
C. Anti-Jo I autoantibodies
D. A & B
E. A & C
What autoimmune disorder is most commonly associated with the case above?
A. Systemic lupus erythematosus
B. Rheumatoid arthritis
C. Scleroderma
D. Polymyositis
E. Hashimoto's thyroiditis
Which of the following is TRUE regarding white infarcts?
A. Can be seen in previously congested tissues that become ischemic
B. It is seen when perfusion to organs with dual blood supply is compromised
C. It is seen with venous occlusion
D. Occur in solid organ infarcts
E. All of the above

A 45 year old female gave birth to a term baby boy with flat facial profile, oblique palpebral fissures, epicanthal folds, and
single palmar crease. Which of the following can be said of his condition?
A. Most common cause is translocation of the long arm of chromosome 21 to chromosome 22 or 14.
B. Paternal age has a stronger influence than maternal age.
C. It is said to be the leading cause of severe mental retardation.
D. Meiotic nondisjunction as the cause is seen in only 4% of cases.
E. All of the above
The case above has a 10 to 20-fold increased risk of what malignancy?
A. Acute Leukemia
B. Rhabdomyosarcoma
C. Wilm's tumor
D. Neuroblastoma
E. Osteosarcoma
A baby boy is born with low set ears, short neck, micrognathia, overlapping fingers, congenital heart defects, renal
malformations, with limited hip abduction and rocker-bottom feet. What genetic condition does he most likely have?
A. Down Syndrome
B. Patau Syndrome
C. Fragile X Syndrome
D. Edwards Syndrome
E. Turner Syndrome

40 year old male consulted you because of urinary hesitancy, intermittency, and poor urinary stream. Digital rectal exam
showed an enlarged prostate. What feature DOES NOT describe benign prostatic hyperplasia?
A. Nodules are composed of variable proportions of proliferating glandular elements and fibromuscular stroma.
B. Nodules may appear solid or contain cystic spaces.
C. The glands are lined by a single uniform layer of cuboidal or low columnar epithelium, lacking basal cell layer.
D. Hyperplastic glandular and stromal elements may project into the bladder lumen as a pedunculated mass.
E. None of the above
Suppose that you requested for serum prostate specific antigen (PSA) levels. What is the cut-off value to say that it is
elevated?
A. 4 ng/mL
B. 5 ng/mL
C. 6 ng/mL
D. 8 ng/mL
E. 10 ng/mL

12 year old boy was brought to your clinic due to a painful firm enlarging mass on his right knee. Radiographs show a large,
destructive, mixed lytic and blastic mass with indictinct infiltrating margins that breaks through the cortex and lifts the
periosteum, showing a characteristic triangular shadow. What is the most likely diagnosis?
A. Chondrosarcoma
B. Giant cell tumor
C. Osteochondroma
D. Ewing's sarcoma
E. Osteosarcoma
14 year old boy was brought to your clinic due to painful firm enlarging mass on his right thigh. Imaging studies show a
destructive lytic tumor with infiltrative margins and extension into surrounding soft tissues with characteristic periosteal
reaction with deposition of bone in an onion-skin pattern. What is the most likely diagnosis?
A. Chondrosarcoma
B. Giant cell tumor
C. Osteochondroma
D. Ewing's sarcoma
E. Osteosarcoma
Which of the following is NOT related to the pathogenesis of cirrhosis?
A. Death of hepatocytes
B. Extracellular matrix deposition
C. Vascular reorganization
D. Diffuse fibrosis
E. None of the above
Which of the following clinical phases of a vulnerable plaque seen in atherosclerotic disease can occur with mural
thrombosis, embolization and wall weakening?
A. Aneurysm and rupture
B. Occlusion by thrombus
C. Critical stenosis
D. Obliterative endarteritis
E. Dystrophic calcification
Which of the following clinical phases of a vulnerable plaque seen in atherosclerotic disease can occur with plaque rupture,
erosion, hemorrhage, mural thrombosis and embolization?
A. Aneurysm and rupture
B. Occlusion by thrombus
C. Critical stenosis
D. Obliterative endarteritis
E. Dystrophic calcification
Which of the following clinical phases of a vulnerable plaque seen in atherosclerotic disease can occur with progressive
plaque growth?
A. Aneurysm and rupture
B. Occlusion by thrombus
C. Critical stenosis
D. Obliterative endarteritis
E. Dystrophic calcification
Which of the following is NOT TRUE regarding myocardial response to ischemia?
A. The earliest biochemical change is loss of aerobic glycolysis and consequent lactic acid accumulation in the cardiac
myocytes
B. The functional consequence is a rapid loss of contractility.
C. Myofibrillar relaxation, glycogen depletion, cell and mitochondrial swelling are reversible.
D. Severe ischemia lasting for at least 20 to 40 minutes can still be reversible.
E. None of the above
Which heart valve is most commonly affected in Libman-Sacks endocarditis?
A. Mitral valve
B. Aortic valve
C. Tricuspid valve
D. Pulmonic valve
E. Valvular involvement in Systemic Lupus Erythematosus is nil

PHARMACOLOGY
Which of the following regarding Fluconazole is TRUE?
A. It is the azole of choice in the treatment and secondary prophylaxis of cryptococcal meningitis
B. It requires low gastric pH for absorption
C. It has activity against Candida, Histoplasma and Aspergillus species
D. Drug interaction is relatively more common than Itraconazole
E. All of the above
Absorption and bioavailability of which of these drugs is NOT affected by food intake?
A. Cloxacillin
B. Cefuroxime
C. Amoxicillin
D. A & B
E. A & C
Which of the followiong drugs is a cytochrome p450 inhibitor?
A. Rifampicin
B. Phenytoin
C. Ketoconazole
D. St. John's Wort
E. Carbamazepine
A patient had just given birth via normal spontaneous delivery. There is significant uterine bleeding and the uterus was not
well-contracted. Which of the following drugs should be given only at the time or soon after delivery of the placenta?
A. Ergonovine
B. Ergotamine
C. Oxytocin
D. Methylsergide
E. Pergolide
Why is magnesium hydroxide given in combination with aluminum hydroxide in antacid formulations?
A. Aluminum hydroxide increases the bioavailability of magnesium hydroxide, responsible for the acid-neutralizing activity.
B. Magnesium hydroxide when in contact with gastric acid releases toxic metabolites that are neutralized by the
aluminum.
C. Combination decreases the chance of developing potential side effects.
D. Combination acts synergistically to inhibit the H+-K+ ATPase pump of the parietal cells.
E. Aluminum when not given in combination with magnesium hydroxide passes quickly through the gastrointestinal tract
unchanged.
Which of the following antineoplastic agents may cause pulmonary fibrosis?
A. CIsplatin
B. Doxorubicin
C. Etoposide
D. Cyclophosphamide
E. Busulfan

Which of the following is an irreversible inhibitor of the aromatase enzyme?


A. Anastrazole
B. Letrozole
C. Exemestane
D. Finasteride
E. Tamoxifen
Which antihypertensive drug can cause glucose intolerance, hyperlipidemia and hypercalcemia?
A. Furosemide
B. Acetazolamide
C. Hydrochlorthiazide
D. Hydralazine
E. Methyldopa
Which antibiotic acts by binding firmly to the D-Ala-D-Ala terminus of nascent peptidoglycan pentapeptide inhibiting bacterial
cell wall synthesis?
A. Meropenem
B. Dalfopristin
C. Vancomycin
D. Aztreonam
E. Linezolid

Which of the following combination of drugs show syngergistic interaction?


A. Salbutamol + ipratropium
B. Trimethoprim + sulfamethoxazole
C. Naloxone + morphine
D. Probenecid + penicillin
E. Diphenhydramine and diazepam
Which anti-asthma drug when used frequently alone can lead to tachyphylaxis or tolerance?
A. Methylxanthine
B. Anticholinergic agent
C. Beta-2 agonist
D. Inhaled corticosteroid
E. Leukotriene receptor antagonist
A 16 year old female developed hirsutism, coarsening of facial features and gingival hyperplasia after being maintained on
an antiepileptic drug. What is the most likely drug involved?
A. Carbamazepine
B. Levetiracetam
C. Valproic acid
D. Phenobarbital
E. Phenytoin

SURGERY
An athlete while playing basketball twisted his right knee and heard a popping sound, with associated sharp pain and
sudden "give" of the knee. You suspected an anterior cruciate ligament tear. You assessed this by pulling the tibia forward
while the knee is stabilized at 20-30 degrees of flexion. What special test did you just do?
A. Lachman test
B. Anterior drawer test
C. Posterior drawer test
D. Valgus stress test
E. Varus stress test
A 19 year old consulted you for vague right shoulder pain, fatigue and heaviness of his right arm that is worse when he is
wearing his backpack. You suspect thoracic outlet syndrome. You palpate the patient's radial pulse on the right and ask the
patient to extend and rotate his head to the same side, as if looking over his shoulder. You then extend and externally rotate
his right arm and ask him to take and hold a deep breath while still feeling the radial pulse. What maneuver was done?
A. Adson's test
B. Allen test
C. Ross stress test
D. Wright's test
E. Military brace test

Thoracic outlet syndrome most commonly involves compression of which artery?


A. External carotid
B. Subclavian
C. Axillary
D. Brachial
E. Vertebral
Which statement describes a Piedmont fracture?
A. Fracture of the distal radius with dislocation of the distal radioulnar joint
B. Fracture of the radial styloid process
C. Fracture of the distal radius with the fragment displaced dorsally
D. Fracture of the distal radius with the fragment displaced ventrally
E. Fracture of the ulna with dislocation of the radial head
Hyperextension injuries of the spine can lead to which spinal cord syndrome?
A. Anterior cord syndrome
B. Posterior cord syndrome
C. Central cord syndrome
D. Brown-Sequard syndrome
E. Dandy-walker syndrome
You ask the patient to grasp a piece of paper between his right thumb and index finger as you attempt to pull it away. You
noted that the patient flexed the distal phalanx of his right thumb to keep hold of the paper. What sign was elicited?
A. Jeanne's sign
B. Phalen's sign
C. Froment's sign
D. Ninhydrin sign
E. Wrinkle sign
Which peripheral nerve in the upper extremity is being tested with the above test?
A. Median nerve
B. Ulnar nerve
C. Radial nerve
D. Musculocutaneous nerve
E. Axillary
What is the most consistent sign of acute appendicitis during pregnancy?
A. Nausea and vomiting
B. Anorexia
C. Fever
D. Pain in the right side of the abdomen
E. Dysuria
A 60 kg 40 year old male suffered superficial partial thickness burns over his whole right upper arm, anterior chest, anterior
aspect of his right lower extremity and genital area. How many milliliters(mL) of lactated ringers solution should be infused in
the first 8 hours following the Parkland formula?
A. 3360 mL
B. 3420 mL
C. 4420 mL
D. 6720 mL
E. 6840 mL

INTERNAL MEDICINE
Which of the following laboratory parameters is NOT consistent with iron deficiency anemia?
A. Peripheral blood smear shows microcytic hypochromic red blood cells
B. Serum iron <30 ug/dL
C. Serum ferritin <15 ug/L
D. TIBC <360 ug/dL
E. Transferrin saturation of <10%
The goal of therapy in individuals with iron-deficiency anemia is not only to repair anemia, but also to provide stores of at
least 0.5-1g of iron. Treatment shoud be sustained for how long in order to replenish iron stores in the body?
A. 2 - 4 weeks
B. 1- 3 months
C. 3- 6 months
D. 6- 12 months
E. 12- 18 months
Which is the most commonly observed laboratory finding in patient's with acute pancreatitis?
A. Leukocytosis
B. Serum amylase 1,5x elevated
C. Serum lipase 1.5x elevated
D. Hypocalcemia
E. Elevated BUN
A 20 year old female consulted you due to multiple erythematous papules and rounded plaques covered with silvery
micaceous scales located at the elbows, knees, gluteal areas and scalp. Which of the following is NOT considered a risk
factor for her condition?
A. Stress
B. Infection
C. Lithium
D. UV exposure
E. Antimalarial drugs
Suppose that the patient in the above case did not manifest the classic skin findings, where else in the body can you look
for signs that could point toward the diagnosis?
A. Nails
B. Teeth
C. Lips
D. Sacrum
E. Hair
A 22 year old female consulted you because of a red patch evolving to a reddish-purplish target lesion over her upper lip
that appeared 2 hours after taking paracetamol for her headache. She denies having difficulty breathing, abdominal pain
and pruritus. No other lesions were noted as well. She verbalized that this same reaction occurred last month, in the same
area in the upper lip, when she also took the same medication, that resolved spontaneously. What is the most likely
diagnosis?
A. Erythema multimforme minor
B. Erythema multiforme major
C. Stevens-Johnson syndrome
D. Fixed drup eruption
E. Toxic epidermal necrolysis

Confirmation of your diagnosis in the above case can be done by:


A. Drug provocation test
B. Radioallergosorbent test
C. Skin biopsy
D. Desensitization test
E. All of the above
A 40 year old garbage collector, has been having chronic burning epigastric pain, which would occassionally awaken him in
the early morning. For the past few days, he has been having black tarry stools. Where is the most likely source of
gastrointestinal bleeding?
A. Esophagus
B. Stomach
C. Duodenum
D. Sigmoid colon
E. Anus
Presence of melena or black, tarry, foul-smelling stool, which usually implies an upper gastrointestinal bleed, means that
blood has been in the gastrointestinal tract for at least how many hours?
A. 8 hours
B. 12 hours
C. 14 hours
D. 18 hours
E. 24 hours

Which of the following is NOT TRUE of HIV-associated dementia?


A. It is generally a late complication of HIV and can be seen in patients with CD4+ T cell counts >350 cells/uL
B. It is a form of "cortical" dementia hence aphasia, apraxia and agnosia are common
C. Patients may have motor abnormalities such as tremors and dysdiadochokinesia
D. Behavioral changes may occur even without significant changes in level of alertness
E. None of the above
A 24 year old primigravid has just been diagnosed with gestational diabetes mellitus. What can you say to her regarding her
condition?
A. She has a substantial risk (35–60%) of developing DM in the next 10–20 years
B. She has to start immediately with insulin to control her blood sugar levels
C. Start preparing for a caesarian section
D. Diet and exercise have not been proven to lower bood glucose levels in GDM
E. Complications relating to GDM are minimal

OB-GYNE
A 24 year old G1P0, at 7-8 weeks age of gestation was brought to the emergency room due to moderate vaginal bleeding
and passage of meaty tissues. Cervix was 2 cm dilated with palpable tissue remnants at the os. What is the most likely
diagnosis?
A. Incomplete abortion
B. Threatened abortion
C. Complete abortion
D. Missed abortion
E. Inevitable abortion

What is the best course of management?


A. Oxytocin
B. Expectant management
C. Prostaglandin E1
D. Curettage
E. Methylergonovine

Sonographic fetal anatomic survey is best done at what age of gestation?


A. 16-18 weeks
B. 18-20 weeks
C. 20-22 weeks
D. 22-24 week
E. 24-26 weeks

Elevated levels of which of the following has been considered a possible marker for impending preterm labor?
A. Maternal estriol
B. Amniotic fluid AFP
C. Fetal fibronectin
D. D dimer
E. Maternal HCG
What is the first sign histologically that ovulation has taken place?
A. Release of glycoprotein and mucopolysacchardie content by the endometrial cells into the lumen
B. Glandular cell mitosis ceases
C. Stroma becomes edematous
D. Glycogen accumulates in the basal portion forming subnuclear vacuoles
E. Extensive coiling of the endometrial glands
What explains the polyhydramnios associated with severe central nervous system anomalies such as anencephaly?
A. Due to associated gastrointestinal defects
B. Due to impaired swallowing reflex
C. Due to leakage of cerebrospinal fluid into the amniotic fluid
D. Due to excessive urination brought about by dysregulation of bladder control
E. Due to apparent decrease in fetal to amniotic fluid ratio
A 35 year old primigravid at 39-40 weeks age of gestation is in labor. Internal exam showed cervix dilated at 8 cm, fully
effaced, ruptured membranes, station +1. Intrapartum monitoring showed late decelerations. What is its clincal significance?
A. Uteroplacental insufficiency
B. Cord compression
C. Head compression
D. Cord prolapse
E. Uterine atony
A 17 year old sexually active female consulted because of vulvar lesions. Pelvic exam shows multiple cauiliflower-like
lesions on the posterior fourchette and perianal region. What human papilloma virus types can cause this?
A. Types 33 and 35
B. Types 31 and 33
C. Types 16 and 18
D. Types 6 and 11
E. Typer 1 and 6
Which human papilloma virus type is most highly associated with cervical cancer?
A. Type 6
B. Type 11
C. Type 16
D. Type 18
E. Type 31
A 25 year old nulligravid consulted you because of vulvar pain of 3 days duration. On pelvic examination, there is a 4x5cm
tender, cystic, fluctuant mass on the left lower lateral aspect of the majora (4 o'clock position). What is the most likely
diagnosis?
A. Bartholin's gland abscess
B. Inclusion cyst
C. Genital warts
D. Mesonephric cyst
E. Skene duct cyst
What is the definitive management for the above case?
A. Broad spectrum antibiotics & hot sitz bath
B. Cryotherapy
C. Cauterization
D. Marsupialization
E. Excision

Maternal serum hCG peaks at what age of gestation?


A. 4-6 weeks
B. 6-8 weeks
C. 7-9 weeks
D. 12-14 weeks
E. 14-16 weeks

PEDIATRICS
Varicella vaccine can be given as early as what age?
A. 6 months
B. 9 months
C. 12 months
D. 14 months
E. 18 months
A 10 month old infant male was brought to the emergency room due to seizures with associated fever and loose watery
stools. History revealed that the infant has already been confined multiple times for recurrent pneumonia in the past. Patient
would also have frequent productive cough, colds and ear infections. On physical examination, there was minimal to no
tonsillar tissue present and no lymph nodes could be appreciated. If you are considering a primary immunodeficiency, what
is the most likely cause?
A. X-linked agammaglobulinemia
B. Combined variable immunodeficieny
C. Selective IgA deficiency
D. Selective IgM deficiency
E. X-linked lymphoproliferative syndrome

Which of the following is NOT TRUE regarding Bruton agammaglobulinemia?


A. Levels of natural antibodies to type A and B red blood cell polysaccharide antigens (isohemagglutinins) and antibodies
to antigens given during routine immunizations are normal.
B. Flow cytometry is the test of choice to demonstrate the absence of circulating B cells.
C. Serum concentrations of IgG, IgA, IgM, and IgE are very low, usually with total immunoglobulins <100 mg/dL.
D. Growth hormone deficiency can also be observed.
E. None of the above

A 5 year old male was brought to the emergency room due to fever and several bouts of water diarrhea for the past 2 days,
with weakness and decease in appetite. History revealed that the patient has already been confined multiple times for
recurrent pneumonia in the past. Patient would also have frequent productive cough, colds and ear infections. On physical
examination, tonsils and lymph nodes were enlarged. If you are considering a primary immunodeficiency, what is the most
likely cause?
A. X-linked agammaglobulinemia
B. Combined variable immunodeficieny
C. Selective IgA deficiency
D. Selective IgM deficiency
E. X-linked lymphoproliferative syndrome

What is the treatment of choice in the above case?


A. Antibiotics to treat infections
B. IVIG or SCID at a dose of 400mg/kg per month
C. Stem cell transplantation
D. A & B
E. All of the above
A 2 month old female presents with persistent pneumonia and diarrhea progressing into sepsis. Lymph nodes, tonsils,
adenoids are absent. The thymus is very small. Patient is also lymphopenic with extremely low to absent T lymphocytes and
serum antibodies. The affected child also lack the ability to reject foreign tissue and is at risk for severe or fatal graft-versus-
host disease. If you are considering a primary immunodeficiency, what is the most likely cause?
A. Combined variable immunodeficieny
B. Wiskott-Aldrich syndrome
C. Severe combined immunodeficiency
D. Job's Syndrome
E. DiGeorge syndrome

A term baby was delivered via vacuum-assisted extraction. You immediately notice a fluctuating mass that straddles cranial
sutures or fontanels that increases in size progressively. Most likely, this mass is brought about by a/an:
A. Caput succadeneum
B. Cephalohematoma
C. Subgaleal hemorrhage
D. Depression fracture
E. Epidural hemorrhage
Which is NOT TRUE of a cephalohematoma?
A. No discoloration of the overlying scalp occurs
B. Swelling is not usually visbible for several hours after birth because subperiosteal bleeding is a slow process
C. The lesion becomes a firm tense mass with a palpable rim localized over 1 area of the skull
D. There is diffuse, ecchymotic, edematous swelling of the soft tissues of the scalp involving the area presenting during
vertex delivery
E. It requires no treatment and resorbs within 2 weeks to 3 months

A 2 month old male developed recurrent nonbilious projectile vomiting for the past 5 days that usually occurs after feeding.
A firm, movable, olive shaped mass is palpated above and to the right of the umbilicus beneath the liver edge, with visible
peristaltic waves across the abdomen. What is the most likely diagnosis?
A. Malrotation
B. Hypertrophic pyloric stenosis
C. Intussesception
D. Mesenteric adenitis
E. Duodenal stenosis
(Items 214-216) A 9 month old male developed high grade fever associated with mild injection of the palpebral conjunctivae
and enlarged suboccipital nodes. On defervescence, there was appearance of a faint pink or rose-colored, nonpruritic, 2-
3mm morbiliform rash on the trunk spreading to the face and extremities. What is the most likely diagnosis?
A. Rubella
B. Measles
C. Varicella
D. Erythema infectiosum
E. Roseola infantum
You examined the child's mouth and noted ulcers at the uvulopalatoglossal junction. What do you call these?
A. Forchheimer spots
B. Koplik spots
C. Nagayama spots
D. Café-au-lait spots
E. Pastia lines

What is the etiologic agent?


A. HHV 6
B. HHV 7
C. HHV 8
D. A & B
E. A & C

Oral antiviral therapy in patients who are at increased risk for moderate to severe varicella infection should ideally be given
within how many hours of onset of the exanthem?
A. 12 hours
B. 24 hours
C. 36 hours
D. 48 hours
E. 72 hours
A 7 year old female thalassemia patient suddenly developed fever, malaise, lethargy, profound anemia, including pallor,
tachycardia, and tachypnea. Which of the following pathogens can be a trigger for transient aplastic crisis?
A. Parvovirus B19
B. Coxsackie A virus
C. HHV 8
D. Rubeola
E. Adenovirus

PREVENTIVE MEDICINE
Which republic act, also knows as the Medical Act of 1959, governs the standardization and regulation of medical
education, the examination for registration of physicians, and the supervision, control and regulation of the practice of
medicine in the Philippines?
A. Republic Act No. 2382
B. Republic Act No. 4688
C. Republic Act No. 6675
D. Republic Act No. 7719
E. Republic Act No. 9288
Which republic act, also known as the National Blood Sevices Act of 1994, promotes voluntary blood donation providing for
an adequate supply of safe blood, regulates the blood banks and provides penalties for violation thereof in the Philippines?
A. Republic Act No. 2382
B. Republic Act No. 4688
C. Republic Act No. 6675
D. Republic Act No. 7719
E. Republic Act No. 9288
A study was conducted to test a new diagnostic examination to detect lung cancer. There were 200 participants and 80 of
them had lung cancer. Using the new diagnostic examination, it detected 70 of those with lung cancer. However, 50 of
those who did not have lung cancer also tested positive. What is the sensitivity of the new diagnostic examination?
A. 40.00%
B. 62.50%
C. 71.43%
D. 87.50%
E. 88.15%

What is the specificity of the new diagnostic examination in the previous question?
A. 41.67%
B. 58.33%
C. 66.67%
D. 87.50%
E.
If the88.15%
mean score of 100 students is 80 with a standard deviation of 5 in a normal distribution, what percentage of students
will have a score of more than 90%?
A. ~2.5%
B. ~5%
C. ~7.5%
D. ~10%
E. ~12.5%
What is the coefficient of variation of the above set data?
A. 5.56%
B. 6.25%
C. 8.14%
D. 9.78%
E. 10.5%
The following increases prevalence rate of a given disease EXCEPT:
A. In-migration of susceptible individuals
B. Out-migration of healthy individuals
C. High case fatality rate
D. Decreased cure rate without mortality
E. None of the above
Which of the following is NOT TRUE of secondary prevention?
A. It involves early diagnosis and prompt treatment
B. The aims is to reduce the impact of a disease or injury that has already occurred
C. Its target population are individuals who are still asymptomatic
D. It seeks to reduce prevalence of a disease
E. None of the above
Which of the following is NOT TRUE of cross-sectional studies?
A. It utilizes survey forms to establish incidence or prevalence of a disease
B. It can be extended to a case control or a cohort study
C. It can establish association or relationship between diseases and other variables of interest
D. It can
A study wasbedone
bothto
aninvestigate
analytical and a descriptiveofstudy
the relationship cigarette smoking and gastric cancer. The total population was 1000 and
E. None of the above
650 among them were smokers. The participants were followed up for 25 years. At the end of the study, 150 among the
smokers and 20 among the nonsmokers developed gastric cancer. What type of analytical study was employed?
A. Cross-sectional study
B. Case control study
C. Cohort study
D. Case series
E. Ecologic study
What is the relative risk of developing gastric cancer among smokers to non-smokers in the above data?
A. 4.04
B. 5.71
C. 8.56
D. 10.12
E.
A 4023.08
year old male suffered bilateral femoral fractures secondary to a motor-vehicular accident. His inability to climb the
stairs to his 4th floor office for work is considered a/an:
A. Impairment
B. Abnormal function
C. Activity limitation
D. Disability
E. Handicap
This refers to the ability of an agent to produce serious illness and is measured in terms of fatality:
A. Infectivity
B. Pathogenicity
C. Virulence
D. Immunogenicity
E. Antigenicity
There were a total of 120 attendants in a highschool reunion party. 80 of the attendants ate the macaroni salad, 95 ate the
chicken fingers, and 110 ate the tuna tartare. 55 of the attendants who ate the macaroni salad were brought to a hospital
due to vomiting and diarrhea. What is the attack rate in this case?
A. 45.83%
B. 50%
C. 57.89%
D. 68.75%
E. 74.89%
PROPOSE
D ANSWER

SIMILAR TO PREVIOUS BOARD EXAM CONCEPT/PRINCIPLE.


Tryptophan is the precursor of niacin, serotonin and melatonin.
Melanin is synthesized from tyrosine, along with L-dopa, dopamine,
A norepinephrine, epinephrine and the thyroid hormones. Ref: Topnotch
handouts

A
SIMILAR TO PREVIOUS BOARD EXAM CONCEPT/PRINCIPLE.
Denaturation DOES NOT involve breaking down of the peptide bonds

A of proteins. The rest of the statements are TRUE. Ref: Topnotch


handouts

SIMILAR TO PREVIOUS BOARD EXAM CONCEPT/PRINCIPLE.


Transamination of aspartate yields oxaloacetate and glutamate. Ref:

C Topnotch handouts

SIMILAR TO PREVIOUS BOARD EXAM CONCEPT/PRINCIPLE. A


refers to Vmax. B refers to Km. C refers to turnover number. D refers

B to catalytic constant. E refers to catalytic efficiency. Ref:Harper's


Illustrated Biochemistry 30th edition.

SIMILAR TO PREVIOUS BOARD EXAM CONCEPT/PRINCIPLE.


Michaelis Menten Equation shows the relationship between initial
reaction velocity and subtrate concentration. Lineweaver-Burk plot is
the double reciprocal plot of the equation. Eadie-Hofstee Plot depicts
B velocity as a function of the ratio between velocity and substrate
concentration. Hanes-Woolf Plot depicts the ratio of substrate
concentration and velocity as a function of substrate concentration. Hill
Plot describes the behavior of enzymes that exhibit cooperative
binding of a substrate. Ref:Harper's Illustrated Biochemistry 30th
edition.
SIMILAR TO PERVIOUS BOARD EXAM CONCEPT/PRINCIPLE. Ref:
Topnotch handouts

C
SIMILAR TO PREVIOUS BOARD EXAM CONCEPT/PRINCIPLE.
Complex I is inhibited by barbiturates, piercidin A, amytal and

D rotenone. Complex II is inhibited by malonate, carboxin and TTFA.


Complex III is inhibited by antimycin A and dimercaprol. Complex IV is
inhibited by cyanide, carbon monoxide, sodium azide and hydrogen
sulfide. Ref: Topnotch handouts
SIMILAR TO PREVIOUS BOARD EXAM CONCEPT/PRINCIPLE.
Ref:Harper's Illustrated Biochemistry 30th edition.

SIMILAR TO PREVIOUS BOARD EXAM CONCEPT/PRINCIPLE.


GLUT 1 is found in brain, kidney, colon, placenta, RBCs. GLUT 2 is

D found in the liver, pancreas, small intestines, kidney. GLUT 3 is found


in brain, kidney and placenta. GLUT 4 requires insulin for glucose
uptake and is found in muscle and adipose tissue. GLUT 5 mediates
absorption of fructose in the small intestines. Ref: Topnotch handouts
SIMILAR TO PREVIOUS BOARD EXAM CONCEPT/PRINCIPLE. Ref:
Topnotch handouts

A
SIMILAR TO PREVIOUS BOARD EXAM CONCEPT/PRINCIPLE. Ref:
Topnotch handouts

Pyruvate dehydrogenase enzyme complex which catalyzes the


irreversible oxidative decarboxylation of pyruvate to acetyl coA
requires 5 coenzymes namely thiamine pyrophosphate, FAD, NAD,
coenzyme A, and lipoic acid. Ref:Harper's Illustrated Biochemistry 30th
E edition.

SIMILAR TO PREVIOUS BOARD EXAM CONCEPT/PRINCIPLE. 11-


cis retinal is a component of the visual pigment rhodopsin. Retinoic

C acid regulates growth, development and tissue differentiation. Beta


carotene is an antioxidant. Ref: Topnotch handouts
SIMILAR TO PREVIOUS BOARD EXAM CONCEPT/PRINCIPLE.
Ref:Harper's Illustrated Biochemistry 30th edition.

A
SIMILAR TO PREVIOUS BOARD EXAM CONCEPT/PRINCIPLE.
Athough the liver can synthesize ketone bodies, it cannot utilize them
because it lacks the enzyme succinyl CoA-acetoacetate-CoA
B transferase (thiophorase). Ref:Topnotch handouts

SIMILAR TO PREVIOUS BOARD EXAM CONCEPT/PRINCIPLE. Ref:


Topnotch handouts

C
B

SIMILAR TO PREVIOUS BOARD EXAM CONCEPT/PRINCIPLE. All


of the following are TRUE. Ref: Topnotch handouts

E
SIMILAR TO PREVIOUS BOARD EXAM CONCEPT/PRINCIPLE.
Renal arteries supply the upper third of the ureter. The gonadal

B arteries supply the middle third. The superior vesical arteries supply
the lower third. Ref: Topnotch handouts

SIMILAR TO PREVIOUS BOARD EXAM CONCEPT/PRINCIPLE. Ref:


Topnotch handouts

SIMILAR TO PREVIOUS BOARD EXAM CONCEPT/PRINCIPLE.


Hernia located within the Hesselbach's triangle or medial to the inferior

D epigastric artery is a Direct Inguinal Hernia usually due to weakness of


the abdominal wall. Ref: Topnotch handouts
SIMILAR TO PREVIOUS BOARD EXAM CONCEPT/PRINCIPLE. The
semitendinosus, semimembranosus and biceps femoris muscles are
collectively termed as the hamstring muscles and they serve to extend
the thigh over the hip joint and flex the knee joint. The semitendinosus,
C semimembranosus and long head of the biceps femoris, all of which
crosses the hip joint and in effect extends the hip, are all innervated by
the tibial division of the sciatic nerve. The short head of the biceps
femoris, originating from the femur, is innervated by the common
peroneal division of the sciatic nerve. Ref: Topnotch handouts
SIMILAR TO PREVIOUS BOARD EXAM CONCEPT/PRINCIPLE. The
roof of the orbit is formed by the orbital plate of the frontal bone and by
the lesser wing of the sphenoid. The floor is formed by the orbital
surface of the maxilllary and zygomatic bones as well as a small orbital
D process of the palatine bone. The medial wall is formed by the orbital
plate of the ethmoid, frontal processes of the maxilla, lacrimal bone
and a small portion of the sphenoid. The lateral wall is formed by the
frontal processes of the zygomatic and the greater wing of the
sphenoid.
SIMILAR TO PREVIOUS BOARD EXAM CONCEPT/PRINCIPLE.
Stages of neutrophil development: Myeloblast-> promyelocyte->

D myelocyte-> metamyelocyte-> band form-> neutrophil

SIMILAR TO PREVIOUS BOARD EXAM CONCEPT/PRINCIPLE.


Shaken baby syndrome usually occurs when a parent or caregiver
shakes a baby vigorously out of anger or frustration. The violent
shaking of the baby's head may lead to fatal brain injuries, most
A commonly subdural hematoma. It may also lead to subarachnoid
hemorrhage, diffuse axonal injury, retinal hemorrhages and cervical
spine injuries, and may result to decreased sensorium, vomiting,
seizures, head posturing, cardiorespiratory instability and potentially
death.
SIMILAR TO PREVIOUS BOARD EXAM CONCEPT/PRINCIPLE. Ref:
Topnotch handouts

C
SIMILAR TO PREVIOUS BOARD EXAM CONCEPT/PRINCIPLE. The
afferent and efferent limbs of the gag reflex are supplied by the

D glossopharyngeal and vagus nerves respectively. Ref: Topnotch


handouts

SIMILAR TO PREVIOUS BOARD EXAM CONCEPT/PRINCIPLE. A


describes the interosseous (talocalcaneal) ligament. B describes the
plantar calcaneonavicular ligament or spring ligament. C describes the
bifurcated or Y ligament of the foot. D describes the long plantar
ligament. E describes the short plantar ligament. Ref: Snell Clinical
B Anatomy by Regions 8th edition
SIMILAR TO PREVIOUS BOARD EXAM CONCEPT/PRINCIPLE. The
anterior cardiac vein and venae cordis minimae drain directly into the

A right atrium. The great, small, and middle cardiac veins drain first into
the coronary sinus before draining into the right atrium. Ref: Topnotch
handouts

SIMILAR TO PREVIOUS BOARD EXAM CONCEPT/PRINCIPLE. The


thymus can be found in the superior and anterior mediastinum. The

D rest of the choices can be seen in the posterior mediastinum. Ref:


Topnotch handouts

SIMILAR TO PREVIOUS BOARD EXAM CONCEPT/PRINCIPLE. The


root of the penis/clitoris is located in the superficial perineal space.
Structures found in the deep perineal space include the internal
E pudendal vessels, dorsal nerve of the penis/clitoris, sphincter urethrae
muscle, deep transverse perineal muscle, and the membranous
urethra and bulbourethral gland(Cowper's gland) in the males, and part
of the urethra and vagina in the females. Ref: Topnotch handouts
SIMILAR TO PREVIOUS BOARD EXAM CONCEPT/PRINCIPLE. All
of the following are TRUE. Herring bodies are distended terminations

E of axons of cells in the supraoptic and paraventricular nuclei where


neurosecretory granules, containing oxytocin and vasopressin, are
located. They are seen in the posterior pituitary gland. Ref: Wheater's
Functional Histology
SIMILAR TO PREVIOUS BOARD EXAM CONCEPT/PRINCIPLE.
When a person opens his mouth, the lateral pterygoids contract to pull
the mandible forward, preventing the angle of the jaw from impinging
B on the parotid gland and sternocleidomastoid muscle to facilitate
depression of the mandible/jaw. The masseter, temporalis, and medial
pterygoids elevate the mandible. The depressor labii inferioris pulls the
lower lip inferiorly. Ref: Snell Clinical Anatomy by Regions 8th edition

SIMILAR TO PREVIOUS BOARD EXAM CONCEPT/PRINCIPLE.


VRSA (Vancomycin resistant staphylococcus aureus) gene is carried

B in vanA. MRSA (Methicillin resistant staphylococcus aureus) is carried


in mecA. Ref: Jawetz Microbiology 26th edition.

SIMILAR TO PREVIOUS BOARD EXAM CONCEPT/PRINCIPLE. Ref:


Jawetz Microbiology 26th edition.

B
SIMILAR TO PREVIOUS BOARD EXAM CONCEPT/PRINCIPLE.
Reinfection with RSV is common in both children and adults. Although
reinfections tend to be symptomatic, the illness is usually limited to the
A upper respiratory tract, resembling a cold in healthy individuals. Ref:
Jawetz Microbiology 26th edition.
Choices A to D are true. Take note that recurrent infections with HSV
are LESS extensive and LESS severe. Many are in fact asymptomatic,

E reflected only by viral shedding in secretions. Ref: Ref: Jawetz


Microbiology 26th edition.

SIMILAR TO PREVIOUS BOARD EXAM CONCEPT/PRINCIPLE.


Lymphogranuloma venereum is a sexually transmitted disease caused
by Chlamydia trachomatis and is characterized by suppurative inguinal
adenitis. Ref: Ref: Jawetz Microbiology 26th edition.
E

SIMILAR TO PREVIOUS BOARD EXAM CONCEPT/PRINCIPLE. Ref:


Jawetz Microbiology 26th edition.

A
SIMILAR TO PREVIOUS BOARD EXAM CONCEPT/PRINCIPLE. Ref:
Jawetz Microbiology 26th edition.

B
SIMILAR TO PREVIOUS BOARD EXAM CONCEPT/PRINCIPLE. Ref:
Jawetz Microbiology 26th edition.

C
SIMILAR TO PREVIOUS BOARD EXAM CONCEPT/PRINCIPLE. Ref:
Jawetz Microbiology 26th edition.

SIMILAR TO PREVIOUS BOARD EXAM CONCEPT/PRINCIPLE.


Phospholamban is known to inhibit SERCA therefore inhibition of
phospholamban allows SERCA-mediated uptake of calcium ions into
the sarcoplasmic reticulum causing cardiac muscle relaxation. Also,
E force of contraction is increased because more calcium is available for
release. Downregulating phospholamban has been said to decrease
incidence of dilated cardiomyopathy and is being implicated in the
treatment of heart failure. Ref: Berne and Levy Physiology 7th edition.
SIMILAR TO PREVIOUS BOARD EXAM CONCEPT/PRINCIPLE.
Apraxia refers to the loss of the ability to execute previously learned
activities in the absence of weakness, ataxia, sensory loss, or
extrapyramidal derangement. The patient for example may be unable
D to dram a simple diagram or describe how to get from home to his
work. This often involves a lesion in the dominant hemisphere
particularly the posterior parietal association areas (Brodmann's area 5
& 7) and can also be seen with lesions in the premotor cortex. Ref:
Kaplan USMLE Step 1 2017
SIMILAR TO PREVIOUS BOARD EXAM CONCEPT/PRINCIPLE. In
96% of right-handed individuals, who constitute 91% of the human

A population, the left hemisphere is the dominant or categorical


hemisphere; and the right hemisphere is the non-dominant or the
representational hemisphere. Ref: Ganong Review of Medical
Physiology 23rd edition.
SIMILAR TO PREVIOUS BOARD EXAM CONCEPT/PRINCIPLE.
Lymphocytes are part of the body's defense against cancer. After birth,

E most of the lymphocytes are formed in the lymph nodes, thymus, and
spleen from precursor cells that originally came from the bone marrow.
Lymphocytes are part of the adaptive immune response. Ref: Ganong
Review of Medical Physiology 23rd edition.
SIMILAR TO PREVIOUS BOARD EXAM CONCEPT/PRINCIPLE. This
is referred to as the chloride shift. Bicarbonate ions diffuse out of the
RBCs into the plasma while chloride ions diffuse into the RBCs to
maintain eletrochemical neutrality. Ref: Ganong 23rd edition.
D

SIMILAR TO PREVIOUS BOARD EXAM CONCEPT/PRINCIPLE. The


stomach accommodates the meal by a process of receptive relaxation.
When food enters the stomach, the fundus and upper portion of the
E body relax, this permits an increase in volume without a significant
increase in pressure. Ref: Ganong Review of Medical Physiology 23rd
edition.
SIMILAR TO PREVIOUS BOARD EXAM CONCEPT/PRINCIPLE. Net
force = Forces that favor filtration - Forces that favor reabsorption =
[(capillary hydrostatic pressure + interstitial colloid osmotic pressure) -
(interstitial hydrostatic pressure + capillary colloid osmotic pressure)] =
D [(35+1) - (0+25)] = 11 mmHg out of the capillary or filtration. Ref:
Ganong 23rd edition.

Insulin acts via activation of tyrosine kinase. ANP acts via cGMP.
Cortisol acts via steroid hormone mechanism. PTH acts via cAMP. Ref:

E BRS Physiology

SIMILAR TO PREVIOUS BOARD EXAM CONCEPT/PRINCIPLE.


Situations wherein there is relative lack of oxygen causes a shift to the
RIGHT in the oxygen-hemoglobin dissociation curve, decreasing
D hemoglobin's affinity for oxgen, allowing it to be utilized by peripheral
tissues. Increased CO2, acidity, 2,3 BPG, temperature, high altitude
and exercise cause a shift to the RIGHT in the oxygen-hemoglobin
dissociation curve. Ref: Topnotch handouts
SIMILAR TO PREVIOUS BOARD EXAM CONCEPT/PRINCIPLE. In
Individuals with complete heart block, ventricles beat at a low rate

D independently of the atria, there may also be periods of asystole


lasting a minute or more. The resultant cerebral ischemia causes
dizziness and fainting (Stokes-Adams syndrome). Ref: Ganong Review
of Medical Physiology 23rd edition.

SIMILAR TO PREVIOUS BOARD EXAM CONCEPT/PRINCIPLE. Ref:


www.prc.gov.ph

Icard's test is done by injecting fluorescin subcutaneously and if


circulation is still intact, the whole skin will turn greenish-yellow.
Diaphanous test is done by spreading the fingers apart and shining a
light through the finger webs in which it would appear red in a living
A person, and yellow in the dead. Winslow tets is done by placing a
saucer filled with water or mercury onto the chest or abdomen, and
breathing movement is checked by reflecting artificial light onto the
water and noting whether there would be movement of the image.
Walker test is done to check for presence of gun powder on clothing.
Ref: Topnotch handouts
SIMILAR TO PREVIOUS BOARD EXAM CONCEPT/PRINCIPLE.
Condition for award of moral damages: There must be an injury; there
must be a culpable act; wrong doing act is the proximate cause of the
A injury; award of damages is based on Article 2219 of the Civil Code.
Ref: Topnotch handouts
Ref: Topnotch handouts

B
Ref: Topnotch handouts

Datyloscopy is the art of identification by comparison of fingerprints.


Proscopy is the study of pores found on the papillary friction ridges of

A the skin for purposes of identification. Bibliotics is the science of


handwriting analysis. Graphology is the study of handwriting for the
purpose of determining the writer's personality, character and aptitude.
Ref: Topnotch handouts
Ref: Topnotch handouts

D
SIMILAR TO PREVIOUS BOARD EXAM CONCEPT/PRINCIPLE. Ref:
Topnotch handouts

A doctor whose license was revoked can apply for reinstatement after
2 years. Ref: Topnotch handouts

C
SIMILAR TO PREVIOUS BOARD EXAM CONCEPT/PRINCIPLE. In
this case, the doctor only acted as an ordinary witness because he
was called forth to state the facts perceived during the incident. In
B contrast, an expert witness is called forth to give inferences,
deductions, conclusions or opinions based on facts presented. Ref:
Topnotch handouts

SIMILAR TO PREVIOUS BOARD EXAM CONCEPT/PRINCIPLE.


Failure to obey a subpoena is a ground for indirect contempt of court.
Ref: Topnotch handouts
B
SIMILAR TO PREVIOUS BOARD EXAM CONCEPT/PRINCIPLE. Ref:
Topnotch handouts

E
SIMILAR TO PREVIOUS BOARD EXAM CONCEPT/PRINCIPLE. The
Biological test of Farnum or preciptin test determines whether the

E specimen is of human origin or not. The remaining choices are


microchemical tests for identifying seminal fluid. Ref: Topnotch
handouts

A person who died of a communicable disease must be buried within


12 hours whereas those who died of a non-communicable disease

A must be buried within 48 hours. Ref: Topnotch handouts

SIMILAR TO PREVIOUS BOARD EXAM CONCEPT/PRINCIPLE.


Apoptosis is programmed cell death characterized by cell shrinkage
with nuclear fragmentation. The plasma membrane remains intact and
cellular contents are released via apoptotic bodies, hence no
inflammation would be appreciated. It may either result from a
A physiologic or pathologic stimulus. Necrosis on the other is
characeterized by cellular swelling, pyknosis, karyorrhexis and
karyolysis. Plasma membrane is disrupted and there would be
enzymatic digestion of cellular contents that may leak out of the cell
instigating an inflammatory response. It usually results from a
pathologic stimulus. Ref: Robbins Basic Pathology 9th edition.
SIMILAR TO PREVIOUS BOARD EXAM CONCEPT/PRINCIPLE. Ref:
Robbins Basic Pathology 9th edition.

Sjögren syndrome is an autoimmune disease caused by CD4+ T cell


reactions against unknown antigens in the ductal epithelial cells of the
exocrine glands. There is also systemic B cell hyperactivity, as
D evidenced by the presence of ANAs and rheumatoid factor (RF). Most
patients with primary Sjögren syndrome have autoantibodies to the
ribonucleoprotein (RNP) antigens SS-A (Ro) and SS-B (La). Ref:
Robbins Basic Pathology 9th edition.
SIMILAR TO PREVIOUS BOARD EXAM CONCEPT/PRINCIPLE. In
patients with Sjogren syndrome, rheumatoid arthritis is the most

B commonly associated disorder (~60%). Other associated conditions


include SLE, polymyositis, systemic sclerosis, vasculitis, or thyroiditis.
Ref: Robbins Basic Pathology 9th edition.
SIMILAR TO PREVIOUS BOARD EXAM CONCEPT/PRINCIPLE.
White infarcts occur with arterial occlusions in solid organs with en-
arterial circulations such as the heart, spleen and kidney), and where
tissue density limits the seepage of blood from adjoining patent
vascular beds. Red infarcts on the other hand, occur (1) with venous
occlusions (such as in ovarian torsion); (2) in loose tissues (e.g., lung)
D where blood can collect in infarcted zones; (3) in tissues with dual
circulations such as lung and small intestine, where partial, inadequate
perfusion by collateral arterial supplies is typical; (4) in previously
congested tissues (as a consequence of sluggish venous out ow); and
(5) when flow is reestablished after infarction has occurred (e.g., after
angioplasty of an arterial obstruction). Ref: Robbins Basic Pathology
9th edition.
SIMILAR TO PREVIOUS BOARD EXAM CONCEPT/PRINCIPLE.
Down syndrome is the most common of the chromosomal disorders.
About 95% of affected persons have trisomy 21, the most common
cause of is meiotic nondisjunction. Maternal age has a strong influence
on the incidence of Down syndrome. It occurs in 1 in 1550 live births in
women younger than 20 years, in contrast with 1 in 25 live births in
women older than 45 years. In about 4% of all patients with trisomy 21,
the extrachromosomal material is present not as an extra chromosome
C but as a translocation of the long arm of chromosome 21 to
chromosome 22 or 14. Approximately 1% of patients with trisomy 21
are mosaics, usually having a mixture of 46- and 47-chromosome
cells. The diagnostic clinical features of this condition- flat facial profile,
oblique palpebral ssures, and epicanthic folds are usually readily
evident, even at birth. Down syndrome is a leading cause of severe
mental retardation; approximately 80% of those afflicted have an IQ of
25 to 50. Ref: Robbins Basic Pathology 9th edition.

SIMILAR TO PREVIOUS BOARD EXAM CONCEPT/PRINCIPLE.

A Children with trisomy 21 have a 10- to 20-fold increased risk of


developing acute leukemia. Both acute lymphoblastic leukemias and
acute myeloid leukemias occur. Ref: Robbins Basic Pathology 9th
edition.
SIMILAR TO PREVIOUS BOARD EXAM CONCEPT/PRINCIPLE.
Trisomy 18 or Edwards Syndrome occurs in 1 in 8000 births and
presenet with a prominent occiput, mental retardation, low set ears,
D short neck, micrognathia, overlapping fingers, congenital heart defects,
renal malformation, limited hip abduction and rocker-bottom feet. Ref:
Robbins Basic Pathology 9th edition.

SIMILAR TO PREVIOUS BOARD EXAM CONCEPT/PRINCIPLE. In


BPH, The nodules may appear solid or contain cystic spaces. The
urethra is usually compressed by the hyperplastic nodules, often to a
narrow slit. In some cases, hyperplastic glandular and stromal
elements lying just under the epithelium of the proximal prostatic
urethra may project into the bladder lumen as a pedunculated mass,
C producing a ball-valve type of urethral obstruction.Microscopically the
hyperplastic nodules are composed of variable proportions of
proliferating glandular elements and bromuscular stroma. The
hyperplastic glands are lined by tall, columnar epithelial cells and a
peripheral layer of flattened basal cells. The glandular lumina often
contain inspissated, proteinaceous secretory material known as
corpora amylacea. Choice C actually describes glands seen in
prostatic adenocarcinoma. Ref: Robbins Basic Pathology 9th edition.
SIMILAR TO PREVIOUS BOARD EXAM CONCEPT/PRINCIPLE. In
most laboratories, a serum PSA level of 4 ng/mL is the cutoff between

A normal and abnormal, although some guidelines designate values


above 2.5 ng/mL as abnormal. Ref: Robbins Basic Pathology 9th
edition.

SIMILAR TO PREVIOUS BOARD EXAM CONCEPT/PRINCIPLE.


Osteosarcomas typically manifest as painful enlarging masses,
although a pathologic fracture can be the first sign. Radiographic
imaging usually shows a large, destructive, mixed lytic and blastic
E mass with indistinct infiltrating margins. The tumor frequently breaks
through the cortex and lifts the periosteum, resulting in reactive
periosteal bone formation. A triangular shadow on the x-ray lm
between the cortex and raised periosteum (Codman triangle) is
characteristic of osteosarcomas. Ref: Robbins Basic Pathology 9th
edition.
SIMILAR TO PREVIOUS BOARD EXAM CONCEPT/PRINCIPLE.
Ewing sarcoma/PNET typically manifests as a painful enlarging mass
in the diaphyses of long tubular bones (especially the femur) and the
pelvic at bones. Some patients have systemic signs and symptoms
D suggestive of infection. Imaging studies show a destructive lytic tumor
with in ltrative margins and extension into surrounding soft tissues.
There is a characteristic periosteal reaction with deposition of bone in
an onion-skin pattern. Ref: Robbins Basic Pathology 9th edition.
SIMILAR TO PREVIOUS BOARD EXAM CONCEPT/PRINCIPLE.
Three processes are central to the pathogenesis of cirrhosis: death of

D hepatocytes, extracellular matrix deposition, and


reorganization. Ref: Robbins Basic Pathology 9th edition.
vascular

SIMILAR TO PREVIOUS BOARD EXAM CONCEPT/PRINCIPLE.


There was a series of 3-4 questions asked on Figure 5-14 of Robbins
Basic Pathology 9th edition, page 342 on the clinical complications of
A atherosclerosis. Mural thrombosis, embolization and wall thickening
could lead to aneurysm formation and rupture. Plaque rupture, erosion,
hemorrhage, mural thrombosis and embolization can cause occlusion
by thrombus formation. Progressive plaque growth can cause critical
stenosis.

C
SIMILAR TO PREVIOUS BOARD EXAM CONCEPT/PRINCIPLE. Loss
of the myocardial blood supply leads to profound functional,
biochemical, and morphologic consequences. Within seconds of
vascular obstruction, aerobic glycolysis ceases, leading to a drop in
adenosine triphosphate (ATP) and accumulation of potentially noxious
metabolites (e.g., lactic acid) in the cardiac myocytes. The functional
D consequence is a rapid loss of contractility, which occurs within a
minute or so of the onset of ischemia. Ultrastructural changes
(including myofibrillar relaxation, glycogen depletion, cell and
mitochondrial swell- ing) also become rapidly apparent. These early
changes are potentially reversible. Only severe ischemia lasting at
least 20 to 40 minutes causes irreversible damage and myocyte
death leading to coagulation necrosis. Ref: Robbins Basic Pathology
9th edition.
SIMILAR TO PREVIOUS BOARD EXAM CONCEPT/PRINCIPLE. Ref:
USMLE Step 1 2013 edition.

SIMILAR TO PREVIOUS BOARD EXAM CONCEPT/PRINCIPLE.


Fluconazole is the azole of choice in the treatment and secondary
prophylaxis of cryptococcal meningitis. In contrast to Itraconazole, it
does not require an acidic environment for absorption; it does not have
A coverage for Aspergillus species or other filamentous fungi and drug
interactions are also less common because it has the least effect of all
the azoles on hepatic microsomal enzymes. Ref: Katzung Basic &
Clinical Pharmacology 12th edition.
SIMILAR TO PREVIOUS BOARD EXAM CONCEPT/PRINCIPLE.
Cloxacillin should be taken on an empty stomach (1 hour before or 2

C hours after a meal) because food decreases its absorption. Cefuroxime


should be taken with food to increase its bioavailability. Bioavailability
of amoxicillin is unchanged with meals hence it can be taken with or
without food. Ref: Katzung Basic & Clinical Pharmacology 12th edition.
The rest of the choices are cytochrome p450 inducers. Ref: Topnotch
handouts

C
SIMILAR TO PREVIOUS BOARD EXAM CONCEPT/PRINCIPLE.
Oxytocin is the preferred agent for control of postpartum hemorrhage,
but if this peptide agent is ineffective, ergonovine maleate, 0.2 mg
A given intramuscularly, can be tried. It is usually effective within 1–5
minutes and is less toxic than other ergot derivatives for this
application. It is given at the time of delivery of the placenta or
immediately afterward if bleeding is significant. Ref: Katzung Basic &
Clinical Pharmacology 12th edition.
SIMILAR TO PREVIOUS BOARD EXAM CONCEPT/PRINCIPLE.
Magnesium hydroxide may cause diarrhea and Aluminum hydroxide
may cause constipation. Combination of the two drugs "cancels" out
C and lessens the risk of side effects and offers high neutralizing
capacity. Ref: Katzung Basic & Clinical Pharmacology 12th edition.

SIMILAR TO PREVIOUS BOARD EXAM CONCEPT/PRINCIPLE.


Drugs that can cause pulmonary fibrosis include Busulfan, Bleomycin,

E Bromocriptine, Amiodarone, Nitrofurantoin and Methotrexate. Ref:


Topnotch handouts

SIMILAR TO PREVIOUS BOARD EXAM CONCEPT/PRINCIPLE.


Anastrozole and Letrozole are selective nonsteroidal inhibitors of
aromatase (the enzyme required for estrogen synthesis), They are
effective in some women whose breast tumors have become resistant
C to tamoxifen. Exemestane, a steroid molecule, is an irreversible
inhibitor of aromatase. Like anastrozole and letrozole, it is approved for
use in women with advanced breast cancer. Ref: Katzung Basic &
Clinical Pharmacology 12th edition.
SIMILAR TO PREVIOUS BOARD EXAM CONCEPT/PRINCIPLE. Ref:
Katzung Basic & Clinical Pharmacology 12th edition.

C
SIMILAR TO PREVIOUS BOARD EXAM CONCEPT/PRINCIPLE. Ref:
Katzung Basic & Clinical Pharmacology 12th edition.

SIMILAR TO PREVIOUS BOARD EXAM CONCEPT/PRINCIPLE.


Trimethoprim or pyrimethamine in combination with a sulfonamide
such as sulfamethoxazole blocks sequential steps in folate synthesis,
resulting in marked enhancement (synergism) of the activity of both
B drugs. The combination often is bactericidal, compared with the
bacteriostatic activity of a sulfonamide alone. Choice A & E are
additive. Choice C is antagonism. Choice D is potentiation. Ref:
Katzung Basic & Clinical Pharmacology 12th edition.
SIMILAR TO PREVIOUS BOARD EXAM CONCEPT/PRINCIPLE.
Frequent use of any beta receptor stimulant/agonist may lead to

C tachyphylaxis or the rapid diminution of responsiveness with drug


administration. Ref: Katzung Basic & Clinical Pharmacology 12th
edition.
SIMILAR TO PREVIOUS BOARD EXAM CONCEPT/PRINCIPLE.
Gingival hyperplasia and hirsutism occur to some degree in most
patients treated with phenytoin. Ref: Katzung Basic & Clinical
E Pharmacology 12th edition.

SIMILAR TO PREVIOUS BOARD EXAM CONCEPT/PRINCIPLE.


Lachman test is done by pulling the tibia forward while the knee is
flexed at 20-30 degrees. The anterior drawer test on the other hand is
done while the knee is flexed at 90 degrees. Both tests are done to
B detect ACL pathology and are considered positive when considerable
forward movement of the tibia is noted. Ref: Jones PA-Orthopedic
Specialty Review and Study Guide. 2015.

SIMILAR TO PREVIOUS BOARD EXAM CONCEPT/PRINCIPLE.


Adson's test is done by palpating the patient's radial pulse and asking
the patient to extend and rotate his head to the same side, as if looking
over his shoulder. You then extend and externally rotate his affected
A arm and ask him to take and hold a deep breath while still feeling the
radial pulse. A diminuted or absent radial pulse denotes a positive test
or when the patient complains of paresthesias down the affected arm.
This test is indicative of thoracic outlet syndrome. Ref: Waldman.
Physical Diagnosis of Pain 3rd edition.
SIMILAR TO PREVIOUS BOARD EXAM CONCEPT/PRINCIPLE.
Thoracic outlet syndrome is caused by compression of the
neurovascular structures (subclavian vessels and/or lower trunk or
cords of the brachial plexus) as they exit throught the thoracic outlet.
B Structures can be compressed either within the scalene musculature,
under a congenital band or bony extension of the 7th cervical
transverse process, between the clavicle and the first rib and/or under
the pectoralis minor. Ref: Waldman. Physical Diagnosis of Pain 3rd
edition.
SIMILAR TO PREVIOUS BOARD EXAM CONCEPT/PRINCIPLE.
Piedmont fracture is also known as Galeazzi fracture which is fracture

A of the distal radius with dislocation of the distal radioulnar joint. Ref:
Netter's Musculoskeletal Flash Cards

SIMILAR TO PREVIOUS BOARD EXAM CONCEPT/PRINCIPLE.


Central cord syndrome typically occurs in older persons who

C experience hyperextension injuries. Motor function, pain, and


temperature sensation are preserved in the lower extremities but
diminished in the upper extremities. Some functional recovery usually
occurs, but is often not a return to normal. Ref: Schwartz's Principles of
Surgery 10th
SIMILAR TOedition.
PREVIOUS BOARD EXAM CONCEPT/PRINCIPLE.
Froment's sign is elicited when flexion of the distal phalanx of the
thumb (flexor pollicis longus innervated by the median nerve) is seen
C when attempting to squeeze a sheet of paper between the thumb and
index finger as a compensation for the weakness in thumb adduction
(adductor pollicis, innervated by the ulnar nerve.) Ref: Magee.
Orthopedic Physical Assessment.
SIMILAR TO PREVIOUS BOARD EXAM CONCEPT/PRINCIPLE. see
above explanation.

B
SIMILAR TO PREVIOUS BOARD EXAM CONCEPT/PRINCIPLE.
Appendicitis in pregnancy should be suspected when a pregnant

D woman complains of abdominal pain of new onset. The most


consistent sign encountered in acute appendicitis during pregnancy is
pain in the right side of the abdomen. Ref: Schwartz's Principles of
Surgery 10th edition.
SIMILAR TO PREVIOUS BOARD EXAM CONCEPT/PRINCIPLE.
Whole right upper arm= 9, anterior chest=9, anterior right lower
extremity=9, genital=1, total BSA= 28, hence 28 x 4ml x 60kg=
6720/2= 3360mL. Ref: Topnotch handouts
A

Iron deficiency anemia is characterized by microcytic hypochromic


RBC's, serum iron <30 ug/dL, serum ferritin <15 ug/L, TIBC >360

D ug/dL, transferrin saturation of <10% with normal hemoglobin pattern


on electrophoresis. Ref: Harrison's 19th edition.

SIMILAR TO PREVIOUS BOARD EXAM CONCEPT/PRINCIPLE. The


goal of therapy in individuals with iron-deficiency anemia is not only to
repair the anemia, but also to provide stores of at least 0.5–1 g of iron.
D Sustained treatment for a period of 6–12 months after correction of the
anemia will be necessary to achieve this. Ref: Harrison's 19th edition.

SIMILAR TO PREVIOUS BOARD EXAM CONCEPT/PRINCIPLE.


Leukocytosis (15,000–20,000 leukocytes/μL) occurs frequently in

A patients with acute pancreatitis. Ref: Harrison's 19th edition.

SIMILAR TO PREVIOUS BOARD EXAM CONCEPT/PRINCIPLE.


Psoriasis is an immune-mediated disease clinically characterized by
erythematous, sharply demarcated papules and rounded plaques
covered by silvery micaceous scale. The skin lesions of psoriasis are
D variably pruritic. Traumatized areas often develop lesions of psoriasis
(the Koebner or isomorphic phenomenon). In addition, other external
factors may exacerbate psoriasis, including infections, stress, and
medications (lithium, beta blockers, and anti- malarial drugs). Ref:
Harrison's 19th edition.
SIMILAR TO PREVIOUS BOARD EXAM CONCEPT/PRINCIPLE.
Fingernail involvement, appearing as punctate pitting, onycholysis, nail
thickening, or subungual hyperkeratosis, may be a clue to the
A diagnosis of psoriasis when the clinical presentation is not classic. Ref:
Harrison's 19th edition.

SIMILAR TO PREVIOUS BOARD EXAM CONCEPT/PRINCIPLE.


Clinically, an fixed drup eruption (FDE) begins as a red patch that soon
evolves to an iris or target lesion similar to erythema multiforme, and
may eventually blister and erode. Medications inducing FDEs are
D usually those taken intermittently. Many of the NSAIDs, especially
pyrazolone deriva- tives, paracetamol, naproxen, oxicams, and
mefenamic acid, cause FDE, with a special predilection for the lips.
Sulfonamides, trimethoprim, or the combination are now responsible
for the majority of genital FDEs. Ref: Andrew's 11th edition.
SIMILAR TO PREVIOUS BOARD EXAM CONCEPT/PRINCIPLE. The
diagnosis of FDE is often straightforward and is elucidated by the
history. However, confirmation with provocation tests can be
performed. Provocation tests need to be delayed at least 2 weeks from
A the last eruption. If an oral provocation test is considered, the initial
challenge should be 10% of the standard dose, and patients with
widespread lesions (SJS/TEN-like) should not be challenged. Patch
testing using a drug concentration of 10–20% in petrolatum or water
applied to a previously reacted site is the recommended approach.
Ref: Andrew's 11th edition.
SIMILAR TO PREVIOUS BOARD EXAM CONCEPT/PRINCIPLE. Pain
that awakens the patient from sleep(between midnight and 3AM) is
most discriminating symptom of a duodenal ulcer. Ref: Harrison's 19th
C edition.

SIMILAR TO PREVIOUS BOARD EXAM CONCEPT/PRINCIPLE.


Melena indicates blood has been present in the GI tract for at least 14
h, and as long as 3–5 days. Ref: Harrison's 19th edition.
C

SIMILAR TO PREVIOUS BOARD EXAM CONCEPT/PRINCIPLE. A


major feature of HIV is the development of dementia, defined as a
decline in cognitive ability from a previous level. It may present as
impaired ability to concentrate, increased forgetfulness, difficulty
reading, or increased difficulty performing complex tasks. Initially these
B symptoms may be indistinguishable from findings of situational
depression or fatigue. In contrast to “cortical” dementia (such as
Alzheimer’s disease), aphasia, apraxia, and agnosia are uncommon,
leading some investigators to classify HIV encephalopathy as a
“subcortical dementia” characterized by defects in short-term memory
and executive function. Ref: Harrison's 19th edition.
A

Bleeding that follows partial or complete placental separation and


dilation of the cervical os is termed incomplete abortion. The fetus and
the placenta may remain entirely within the uterus or partially extrude
A through the dilated os. Ref: William's 24th edition.

Unless there is serious bleeding or infection with an incomplete


abortion, any of three options are reasonable—expectant, medical, or
surgical management. Expectant management of spontaneous
D incomplete abortion has failure rates as high as 50 percent. Medical
therapy with prostaglandin E1 (PGE1) has varying failure rates of 5 to
40 percent. Curettage usually results in a quick resolution that is 95- to
100-percent successful. Ref: William's 24th edition.
SIMILAR TO PREVIOUS BOARD EXAM CONCEPT/PRINCIPLE. An
important goal of second and third trimester sonography is to
systematically evaluate fetal anatomy and determine whether specific
anatomical components appear normal or abnormal. If a single
B ultrasound examination is planned for the purpose of evaluating fetal
anatomy, the American College of Obstetricians and Gynecologists
recommends that it be performed at 18 to 20 weeks. At this gestational
age range, complex organs such as the fetal brain and heart can be
imaged clearly enough to visualize many major malformations. Ref:
William's 24th edition.
SIMILAR TO PREVIOUS BOARD EXAM CONCEPT/PRINCIPLE.
Fibronectin detection in cervicovaginal secretions before membrane
rupture is a possible marker for impending preterm labor. It is
measured using an enzyme-linked immunosorbent assay, and values
C exceeding 50 ng/mL are considered positive. Interventional studies
based on the use of fetal fibronectin screening in asymptomatic
women however have not demonstrated improved perinatal outcomes.
The American College of Obstetricians and Gynecologists does not
recommend screening with fetal fibronectin tests. Ref: William's 24th
edition.
By day 17, glycogen accumulates in the basal portion of glandular
epithelium, creating subnuclear vacuoles and pseudostrati cation. is is

D the rst sign of ovula- tion that is histologically evident. It is likely the
result of direct progesterone action through receptors expressed in
glandular cells. Ref: William's 24th edition.

SIMILAR TO PREVIOUS BOARD EXAM CONCEPT/PRINCIPLE.


Severe central nervous system abnormalities, such as anencephaly,

B hydranencephaly, or holoprosencephaly, can result in hydramnios due


to impaired fetal swallowing. Ref: William's 24th edition.
As amnionic fluid pressure increases with uterine contractions,
myometrial pressure exceeds collapsing pressure for vessels coursing
through uterine muscle. This ultimately decreases blood flow to the
A intervillous space. Brief periods of impaired oxygen exchange result,
and if uteroplacental pathology is present, these elicit late fetal heart
rate decelerations. Ref: William's 24th edition.

The patient has genital warts. HPV types 6 and 11 have generally
been recognized as being found most often in benign vulvar warts,
whereas primarily HPV types 16, 18, 31, 33, and 35 are more
D frequently associated with intraepithelial neoplasia or invasive cervical
carcinoma. Ref: Lentz et al Comprehensve Gynecology. 6th edition.

HPV types 16, 18, 31, 33, and 35 are more frequently associated with
intraepithelial neoplasia or invasive cervical carcinoma, but the most

C common type identified in the general U.S. population is type 16, which
is also the type most highly associated with cancer. Ref: Lentz et al
Comprehensve Gynecology. 6th edition.

SIMILAR TO PREVIOUS BOARD EXAM CONCEPT/PRINCIPLE.


Bartholin’s glands are vulvovaginal glands that are located imme-
diately beneath the fascia at about 4 and 8 o’clock, respectively, on the
A posterolateral aspect of the vaginal orifice. The most common large
cystic structure of the vulva is a Bartholin’s duct cyst. This condition
may become painful if the cyst develops into an acute abscess. Ref:
Lentz et al Comprehensve Gynecology. 6th edition.
SIMILAR TO PREVIOUS BOARD EXAM CONCEPT/PRINCIPLE. The
treatment of choice for a symptomatic cyst or abscess is the

D development of a fistulous tract from the dilated duct to the vestibule


termed marsupialization. Ref: Lentz et al Comprehensve Gynecology.
6th edition.

SIMILAR TO PREVIOUS BOARD EXAM CONCEPT/PRINCIPLE.


HCG has been detected in the peripheral blood of the mother as early
as 6 days after ovulation, but it is always seen by day 12. The
concentration doubles every 1.2 to 2 days, reaching its highest point at
7 to 9 weeks of pregnancy. (Lentz et al Comprehensve Gynecology.
C 6th edition.) In William's 24th edition, hCG is detectable in plasma of
pregnant women 7 to 9 days after the midcycle surge of LH that
precedes ovulation. Plasma levels increase rapidly, doubling every 2
days in the first trimester. Peak maternal plasma levels reach
approximately 100,000 mIU/mL between the 60th and 80th days after
menses. At 10 to 12 weeks, plasma levels begin to decline, and a
nadir is reached by approximately16 weeks.

SIMILAR TO PREVIOUS BOARD EXAM CONCEPT/PRINCIPLE.


Varicella vaccine is given subcutaneously at a minimum age of 12

C months. Two doses are recommended. First dose is given at 12-15


months of age. The second dose is usually given at 4-6 years of age.
Ref: Childhood Immunization Schedule 2016 by the PPS, PIDSP, and
PFV.
SIMILAR TO PREVIOUS BOARD EXAM CONCEPT/PRINCIPLE.
Patients with X-linked agammaglobulinemia (XLA), or Bruton agam-
maglobulinemia, have a profound defect in B-lymphocyte
development resulting in severe hypogammaglobulinemia, an absence
of circulating B cells, small to absent tonsils, and no palpable lymph
nodes. Most boys with XLA remain well during the 1st 6-9 mo of life by
virtue of maternally transmitted IgG antibodies. Thereafter, they
acquire infections with extracellular pyogenic organisms, such as
Streptococcus pneumoniae and Haemophilus influenzae, unless they
are given prophylactic antibiotics or immunoglobulin therapy. Infections
A include sinusitis, otitis media, pneumonia, or, less often, sepsis or
meningitis. Infections with Mycoplasma are also particularly
problematic. Chronic fungal infections are seen; Pneumocystis jiroveci
pneumonia rarely occurs. Viral infections are usually handled normally
with the exceptions of hepatitis viruses and enteroviruses. There were
several examples of paralysis when live polio vaccine was
administered to these patients, and chronic, eventually fatal, central
nervous system infections with various echoviruses and
coxsackieviruses have occurred in a significant number of them.
Echovirus-associated myositis resembling dermatomyositis has also
been observed. Ref: Nelson 20th edition.
SIMILAR TO PREVIOUS BOARD EXAM CONCEPT/PRINCIPLE.
Levels of natural antibodies to type A and B red blood cell
polysaccharide antigens (isohemagglutinins) and antibodies to
antigens given during routine immunizations are abnormally low in
XLA, whereas they are normal in transient hypogammaglobulinemia of
infancy. Serum concentrations of IgG, IgA, IgM, and IgE are low,
A usually with total immunoglobulins <100 mg/dL. Growth hormone
deficiency has also been reported in association with XLA.
Neutropenia, responsive to G-CSF, may be present in patients with
XLA. Flow cytometry is an important test to demonstrate the absence
of circulating B cells, which will distinguish this disorder from common
variable immunode ciency, the hyper-IgM syndrome and transient
hypogammaglobulinemia of infancy. Ref: Nelson 20th edition.
SIMILAR TO PREVIOUS BOARD EXAM CONCEPT/PRINCIPLE.
Common variable immunode ciency (CVID) is a syndrome
characterized by hypogammaglobulinemia with phenotypically normal
B cells. It has also been called acquired hypogammaglobulinemia
because of a generally later age of onset of infections. CVID patients
may appear similar clinically to those with XLA in the types of
B infections experienced and bacterial etiologic agents involved, except
that echovirus meningoencephalitis is rare in patients with CVID and
infections are less severe. Patients with CVID often have autoantibody
formation and normal-sized or enlarged tonsils and lymph nodes.
There is often a delay in the diagnosis of more than 5 yr between the
first infections and a definitive diagnosis. Ref: Nelson 20th edition.

Except for the CD40 ligand defect and X-linked lymphoproliferative


syndrome, for which stem cell transplantation is recommended,
judicious use of antibiotics to treat documented infections and regular
D administration of IVIG are the only effective treatments for primary B-
cell disorders. IVIG or SCIG at a dose of 400 mg/ kg per month
achieves trough IgG levels close to the normal range. Higher doses
are indicated in patients with chronic or severe respiratory infections.
Ref: Nelson 20th edition.
SIMILAR TO PREVIOUS BOARD EXAM CONCEPT/PRINCIPLE.
SCID results from mutations in any 1 of at least 13 known genes that
encode components of the immune system crucial for lymphoid cell
development, hence both the B and T cell lineages are deficient. All
patients with SCID have very small thymuses (<1 g) that usually fail to
descend from the neck. Lymph nodes, tonsils, adenoids, and Peyer
patches are absent or extremely underdeveloped. Affected infants
C present within the 1st few months of life with recurrent or persistent
diarrhea, pneumonia, otitis media, sepsis, and cutaneous infections.
Growth may appear normal initially, but extreme wasting usually
ensues after diarrhea and infections begin. Persistent infections with
opportunistic organisms lead to death. Affected infants also lack the
ability to reject foreign tissue and are therefore at risk for severe or
fatal graft -versus- host disease (GVHD). Ref: Nelson 20th edition.

SIMILAR TO PREVIOUS BOARD EXAM CONCEPT/PRINCIPLE. A


subgaleal hemorrhage is a collection of blood beneath the
aponeurosis that covers the scalp and serves as the insertion for the
occipitofrontalis muscle. Bleeding can be very extensive into this large
potential space and may even dissect into the subcutaneous tissues of
the neck. There is often an association with vacuum-assisted
C delivery.This is most likely secondary to rupture of emissary veins
connecting the dural sinuses within the skull with the superficial veins
of the scalp. It manifests as a fluctuating mass that straddles cranial
sutures or fontanels that increases in size after birth. Patients should
be monitored for hypotension, anemia, and the development of
hyperbilirubinemia. These lesions typically resolve over 2-3 wk. Ref:
Nelson 20th edition.
SIMILAR TO PREVIOUS BOARD EXAM CONCEPT/PRINCIPLE.
Choice D describes a caput succadeneum that is diffuse as it may
cross suture lines and extend the midline. The rest of the choices are
D true of cephalohematomas. Ref: Nelson 20th edition.

Nonbilious vomiting is the initial symptom of pyloric stenosis. e


vomiting may or may not be projectile initially but is usually progres-
sive, occurring immediately a er a feeding. Emesis might follow each
feeding, or it may be intermittent. The diagnosis has traditionally been
B established by palpating the pyloric mass. The mass is firm, movable,
approximately 2 cm in length, olive shaped, hard, best palpated from
the left side, and located above and to the right of the umbilicus in the
midepigastrium beneath the liver’s edge. After feeding, there may be a
visible gastric peristaltic wave that progresses across the abdomen.
Ref: Nelson 20th edition.
SIMILAR TO PREVIOUS BOARD EXAM CONCEPT/PRINCIPLE.
Roseola infantum (exanthem subitum, or sixth disease) is an acute,
self-limited disease of infancy and early childhood. It is characterized
by the abrupt onset of high fever, which may be accompanied by
fussiness. The fever usually resolves acutely after 72 hr but may
gradually fade over a day coincident with the appearance of a faint
E pink or rose-colored, nonpruritic, 2-3 mm morbilliform rash on the trunk.
The rash usually lasts 1-3 days but is often described as evanescent
and may be visible only for hours, spreading. from the trunk to the face
and extremities. Because the rash is variable in appearance, location,
and duration, it is not distinctive. Associated signs are few but can
include mild injection of the pharynx, palpebral conjunctivae, or
tympanic membranes and enlarged suboccipital nodes. Ref: Nelson
20th edition.
In Asian countries, ulcers at the uvulopalatoglossal junction
(Nagayama spots) are commonly reported in infants with roseola. Ref:

C Nelson 20th edition.

SIMILAR TO PREVIOUS BOARD EXAM CONCEPT/PRINCIPLE.


Human herpesvirus 6 (HHV-6A and HHV-6B) and human herpesvirus
7 (HHV-7) cause ubiquitous infection in infancy and early childhood.
HHV-6B is responsible for the majority of cases of roseola infantum
D (exanthema subitum or sixth disease) and is associated with other
diseases, including encephalitis, especially in immunocompromised
hosts. A small percentage of children with roseola have primary
infection with HHV-7. Ref: Nelson 20th edition.
SIMILAR TO PREVIOUS BOARD EXAM CONCEPT/PRINCIPLE.
Acyclovir therapy is not recommended routinely by the American
Academy of Pediatrics for treatment of uncomplicated varicella in the
otherwise healthy child. Oral therapy with acyclovir (20mg/kg/dose;
maximum: 800 mg/dose) given as 4 doses/day for 5 days can be used
to treat uncomplicated varicella in individuals at increased risk for
moderate to severe varicella: nonpregnant individuals older than 12 yr
B of age and individuals older than 12 mo of age with chronic cutaneous
or pulmonary disorders; individuals receiving short-term, intermittent, or
aerosolized corticosteroid therapy; individuals receiving long- term
salicylate therapy; and possibly secondary cases among household
contacts. To be most effective, treatment should be initiated as early
as possible, preferably within 24 hr of the onset of the exanthem.
There is less clinical benefit if treatment is initiated more than 72 hr
after onset of the exanthem. Ref: Nelson 20th edition.
The transient arrest of erythropoiesis and absolute reticulocytopenia
induced by parvovirus B19 infection leads to a sudden fall in serum
hemoglobin in individuals with chronic hemolytic conditions. B19-
A induced RBC aplasia or transient aplastic crisis occurs in patients with
all types of chronic hemolysis and/or rapid RBC turnover, including
sickle cell disease, thalassemia, hereditary spherocytosis, and
pyruvate kinase deficiency. Ref: Nelson 20th edition.
SIMILAR TO PREVIOUS BOARD EXAM CONCEPT/PRINCIPLE.
Republic act 2382, also knows as the Medical Act of 1959, governs the
standardization and regulation of medical education, the examination
A for registration of physicians; and the supervision, control and
regulation of the practice of medicine in the Philippines. Ref:
www.lawphil.net/statutes/repacts/ra1959/ra_2382_1959.html

SIMILAR TO PREVIOUS BOARD EXAM CONCEPT/PRINCIPLE.


Republic Act 7719, also known as the National Blood Sevices Act of
1994, promotes voluntary blood donation providing for an adequate
D supply of safe blood, regulates the blood banks, and provides
penalties for violation thereof in the Philippines. Ref:
www.officialgazette.gov.ph/1994/05/05/republic-act-no-7719/

SIMILAR TO PREVIOUS BOARD EXAM


CONCEPT/PRINCIPLE.Sensitivity is number of those that tested
positive in patients with the disease over the total number of diseased
patients=70/80= 87.5% Ref: Topnotch handouts
D

SIMILAR TO PREVIOUS BOARD EXAM CONCEPT/PRINCIPLE.


Specificity is the number of those that tested negative in patients
without the disease over the total number of patients without the
B disease=70/120=58.33% (there were a total of 120 patients without the
disease and it was mentioned that 50 of those that did not have lung
cancer tested positive hence it would mean that the remaining 70 of
the 120 tested negative) Ref: Topnotch handouts
SIMILAR TO PREVIOUS BOARD EXAM CONCEPT/PRINCIPLE. A
score of 90 is 2 standard deviations from the mean as is the score of

A 70 (since the mean is 80 with a standard deviation of +/- 5).


Remember that 95% of observations fall within 2 standard deviations
of the mean such that 5% will fall outside of the 2 standard deviations
where 2.5% is above 90 and 2.5% will be below 70. Ref: Topnotch
handouts
SIMILAR TO PREVIOUS BOARD EXAM CONCEPT/PRINCIPLE.
Coefficient of variation= SD/Mean x 100%= 5/80x100%= 6.25%. Ref:

B Topnotch handouts

High case fatality rate decreases prevalence. The rest of the choices
increases prevalance. Ref: Topnotch handouts

C
SIMILAR TO PREVIOUS BOARD EXAM CONCEPT/PRINCIPLE. All
of the statements describe secondary prevention. Ref: Topnotch

E handouts
All of the statements describe a cross-sectional study. Ref: Topnotch
handouts

E
SIMILAR TO PREVIOUS BOARD EXAM CONCEPT/PRINCIPLE. In
cohort studies, exposed and unexposed populations are identified and

C followed prospectively over time to determine the rate of a specific


clinical disease or event. Ref: Topnotch handouts

SIMILAR TO PREVIOUS BOARD EXAM CONCEPT/PRINCIPLE.


Relative risk= risk of the exposed/risk of the unexposed=

A (150/650)/(20/350)=4.04 Ref: Topnotch handouts

SIMILAR TO PREVIOUS BOARD EXAM CONCEPT/PRINCIPLE.


Handicap is defined as a disadvantage for a given individual, resulting

E from an impairment or disability that limits or prevents the fulfillment of


a role that is normal for that individual. In this example, the bilateral
femoral fractures (impairment) causes the patient inability to climb the
stairs (disability) to go to work in his 4th floor office (handicap). Ref:
Topnotch handouts
Virulence refers to the ability of an agent to produce serious illness and
is measured in terms of fatality. Ref: Topnotch handouts

C
Attack rate= number of persons with the disease/number of persons
exposed to the disease= 55/80=68.75% Ref: Topnotch handouts

D
The diagnosis of a community health problem involves the following, EXCEPT:
A. determining the existence, nature and extent of the problem
B. relating the frequency of the problem to the persons, conditions and other circumstances in the
1 community
C. identifying probable determinants existing in the community
D. recommending strategies/interventions for the solution of the problem

Disease agent responsible for almost up to 50% cases of diarrhea in children ages 6-24 months:
A. Rotavirus
B. Shigella
2
C. Enterotoxigenic E. coli
D. V. cholera

All of the following are necessary to plan and conduct a case-control study, EXCEPT:
A. developing and testing research instruments
B. defining the disease and exposure of interest
3
C. selecting cases and defining a control group
D. determining the duration of the observational (study) period

The addition of nutrients to food in order to to improve the nutritional quality of individual foods of
the total diet of a group,community or population is called:
A. Nutrification
4 B. Fortification
C. Supplementation
D. Prevention

Statistics of disease frequency in a community are usually or routinely obtained from:


A. reports of disease occurrence only
B. registration of deaths only
5
C. morbidity/mortality surveys
D. reports of disease occurrence and registration of death
The objectives of family planning education is difficult to achieved because of the
following,EXCEPT:
A. it has to contend with one of the strongest human urges
B. only negative effects can be demonstrated
6
C. the subject is generally very private to Filipinos
D. many cultural obstacle

Maternal and Child Health activities and services relate to the:


A. well being of the adolescent
B. breast-feeding,nutrition and postnatal care
7
C. supervision and care of pregnant women
D. all of the above

Giardiasis occurs on a worldwide basis and is associated with all of the following,EXCEPT;
A. waterborne transmission
B. asymptomatic carriage
8
C. transmission from person to person
D. invasion of colonic mucosa

Most permanent method for the control of rodent:


A. fumigation
B. natural enemies
9
C. poisons
D. rehabilitation
If the infant mortality of a community is 53/1,000 live births,this means:
A. 53 out of 1,000 persons in the community were infant deaths
B. 53 babies die before reaching their first birthday
10
C. 53 out of 1,000 babies born alive died during infancy
D. 53% of all deaths are infants
The extent to which a particular disease is due to a specific factor can be best measured by:
A. comparing the incidence rates between the exposed and non-exposed
B. comparing for the ratio of the incidence rate among the exposed to those among the non-
exposed
11
C. comparing the difference of the incidence rate of the exposed and non-exposed
D. none of the above

The under five clinic focuses on the following aspect of children:


A. immunization
B. maternal and child care
12
C. weighing and assessment of growth
D. safe water and basic sanitation
The child had been attending day care center. In addition to recommending close surveillance for
early signs of illness,which of the following is the most appropriate management of day care
contacts:
A. no further action
13 B. vaccination of children only
C. vaccination of children and adults
D. antibiotics prophylaxis of adult and children

The following are uses of the growth chart,EXCEPT:


A. to monitor the height of child
B. to monitor the weight of child
14
C. to monitor the vaccination of child
D. to monitor the development of child

The National Drug Policy was established having the following objectives with the exception of:
A. to help ensure the rational use of drugs by the consumers and health professionals
B. to strengthen the capability of the Bureau of Foods and Drugs
15
C. to achieve self-sufficiency in basic drug manufacturing
D. to encourage pharmaceutical research

A disease with long latent period will have an incidence rate of:
A. is gradually increasing with age
B. is uniformly high with all ages
16
C. show no definite pattern
D. is uniformly low for all ages

Which among the following is NOT a method of dispersion:


A. range
B. coefficient of variation
17
C. variance
D. mode

Correct statements concerning fluoridation of drinking water include all of the following,EXCEPT:
A. fluoridation of drinking water reduces caries of about 50 percent
B. the advisability of fluoridation is still controversial among dental public health expert
18
C. the optimum concentration of fluoride in public drinking water depends on the average daily air
temperature of the community served
D. most people cannot taste fluoride at concentrations of 1 part per million (ppm)
The principle involved in the development of multiple barriers in the water supply is the prevention
of:
A. Pollution
B. Contamination
19
C. Dehydration
D. Consumption

Under Primary Health Care, the following are the objectives for an essential health and care
delivery,EXCEPT:
A. national based in scope
20 B. acceptable
C. total community participation
D. accessibility

In any research,one starts with:


A. the production of funds and materials
B. collection of data
21
C. preparation of the study design
D. identification and definition of the problem

The Provincial Hospital X has been having undeclared deaths in the nursery,particularly the
newborns for the past two (2) months. The Director of the Hospital decided to close the nursery
while conducting an investigation. What is the type of action taken by the Director of the Hospital:
22 A. interim
B. corrective
C. preventive
D. adaptive

Guidelines for effective patient education and counseling include all of the following,EXCEPT:
A. obtaining commitment from patients to change behavior
B. involving patients in selection of risk factors that require change
23
C. monitoring progress through follow-up
D. using a single strategy

Which among the attributes of the hosts facilitates the encounter/meeting of the host and the agent
of a disease:
A. habits and customs
24 B. sex
C. age
D. heredity and personality development

A cook is found to be Hepatitis A positive. You should advise that she:


A. receive Hepatitis A vaccine
B. handle only uncooked food
25
C. stop cooking
D. continue cooking but wash hands thoroughly

The most common side effect of a properly administered BCG vaccine:


A. suppurative adenitis
B. convulsion
26
C. Koch's phenomenon
D. abscess formation at the injection site
An investigation done to compare the prevalence of alcohol-intake among liver cancer patient and
lung cancer patient. What type of study is this:
A. Descriptive Study
27 B. Cross-sectional Study
C. Case-control Study
D. Experimental Study

Complications of diabetes mellitus include all of the following,EXCEPT:


A. blindness
B. pancreatic cancer
28
C. limb loss
D. stroke

The following are characteristics features of Epidemiology,EXCEPT:


A. I's methods are generally observational
B. It is a quantitative science
29
C. Its focus is the individual
D. It is an applied science

Which among the following is utilized in the determination of the infant mortality rate:
A. mid-year population
B. total still births in one year
30
C. 0-3 year old population
D. one year total live births

Which of the following statements is /are true of clinical trials:


A. it is usually preventive in purpose
B. randomization answers comparatively of the experimental and control groups with respect to
31 all factors known and unknown
C. like analytic studies,the experimental and control groups are taken from the same population
D. none of the above

The best source of data or distribution according to age,sex and geographical location:
A. registries of certain disease
B. birth certificate
32
C. census
D. reports of occurrence of notifiable disease

Studies in medicine are designed to identify causes of disease. The ultimate goal of such studies is
to alter the frequency or severity of this diseases. All the following are considerations in the
determination of causality,EXCEPT:
33 A. temporal sequence-causative agents must precede their consequence
B. biological gradient-dose response curve
C. concurrency-cause and effect are found at the same time
D. consistency-repeated observations of the same association

One of the following is not considered as a direct cause of maternal death:


A. sepsis
B. toxemia of pregnancy
34
C. hemorrhage
D. cancer
The operating unit of the Department of Health at the local level is:
A. barangay health station
B. puericulture center
35
C. rural health unit
D. regional hospital

What could be a probable indication for rabies vaccination:


A. bites of a healthy dog
B. before a person is bitten by a rabid dog
36
C. licks on abraded skin of a suspicious rabid dog within 14 days of observation
D. indirect contact with a rabid dog

The precursor in the decline of fatality in Europe was:


A. aging population
B. legalization of abortion
37
C. improvement in the standard of living
D. better health care

Which among the following is the primary function of a census:


A. Mortality rates determination
B. Age and sex distribution of the population determination
38
C. Population enumeration
D. Birth rate determination

What is the key to a higher quality health care:


A. Increasing budget for health
B. Better utilization of those already available health workers
39
C. Increasing the number of health workers
D. Providing more nursing service

The following are current health problems,EXCEPT:


A. drug dependence
B. rapid population growth
40
C. communicable disease and malnutrition
D. scarcity of raw materials and funds
The rising trend of cancer incidence and mortality especially in the developing countries is mainly
due to:
A. increasing exposure to carcinogens
B. increasing life span
41
C. better reporting of the disease
D. changing lifestyle

What is the simplest and most economical method of data collection:


A. review of documents
B. observation
42
C. questionnaires
D. query

What is the denominator used in Swaroop's index:


A. Mid-year population
B. Total population age 50 and above
43
C. Total death
D. Total population
In the Philippines, the agency tasked to coordinate all nutrition related activities of the government
and private sectors is the:
A. National Nutrition Council
44 B. DOH's Nutrition Service
C. Nutrition Center of the Philippines
D. Department of Science and Technology
Living bodies that harbor,sustain and maintain the growth and multiplication of the infectious agent
are:
A. chain of transmission
B. etiologic agents
45
C. reservoir of infection
D. causative agents

The most appropriate strategy for teaching the signs and symptoms of dehydration to Community
Health Workers:
A. typewritten handouts on the sign and symptoms of dehydration
46 B. simulation or field observation
C. lectures
D. demonstration-return demonstration

The method for formulating hypothesis based on the similarity and distribution of factors and
47 outcome:
A. method of agreement
B. method of difference
C. method of analogy
D. method of concomitant variation

The incidence of cholelithiasis (gallstones) is increased in all the following, EXCEPT:


A. persons with diabetes
B. persons who are obese
48
C. persons with hypercholesterolemia
D. persons with chronic hemolytic anemia

In rural health clinics,the first contact of patient are the:


A. physicians
B. midwives
49
C. nurses
D. sanitary inspectors

Basic health services means that:


A. there is provision for referral service to higher level of health care
B. there are services without the need for specialist
50
C. these are provided by both medical and paramedical personnel
D. all of the above

Although more stringent in testing hypothesis of cause, Cohort Studies have the following
limitations,EXCEPT:
A. period of follow-up is generally long
51 B. generally expensive
C. generally require large population for study
D. association between suspected cause and disease is under-estimate
One of the cheapest and most useful chemical disinfectant for human excreta is:
A. DDT
B. Lime
52
C. Bichloride
D. Phenol

The development of neuropathic symptoms is associated with chronic exposure to all of the
following substances,EXCEPT:
A. mercury
53 B. lead
C. arsenic
D. sulfur dioxide

One of the following is an indirect method of nutritional assessment:


A. measles mortality rate
B. 1-4 age specific death rate
54
C. percentage of babies with low birth weight
D. greater than 5 years proportionate mortality rate

In a classroom of 25 medical students (15 males and 10 females), 5 males develop hepatitis A over
2-week period. During the next 6 weeks, an additional 3 males and 2 females develop the
infection. The attack rate of hepatitis A in this classroom is:
55 A. 25%
B. 30%
C. 35%
D. 40%
In the Philippines, the principal providers of care during pregnancy and delivery continued to be
the :
A. physicians
B. nurses
56
C. traditional birth attendants
D. midwives

The first available heart for transplantation was availed by the richest patient. Under what principle
can this be justified:
A. medical success
57 B. random selection
C. immediate usefulness
D. medical neediness

Primary Health Care team is composed of:


A. Interdisciplinary Composition of Community
B. Office of Secretary of Health
58
C. Provincial Health Officer and Staff
D. Municipal Health Officer and Staff

The following are examples of secondary level of prevention,EXCEPT:


A. disease detection
B. disease reporting
59
C. specific protection
D. none of the above
A group that is least vulnerable to anemia:
A. pre-schooler
B. lactating woman
60
C. normal neonates up to 3 months
D. pregnant woman

Factors to consider in choosing the method of data collection include the following,EXCEPT:
A. probability of good coverage
B. availability of time and facilities
61
C. acceptability of the procedure/method to the subjects
D. need for personal skill

Because of the high incidence of carriers of this disease, what laboratory examination is requested
for pregnant Filipino women:
A. Rh typing
62 B. Serological test for gonorrhea
C. Hepatitis profile
D. Serological test for syphilis
Parents who abuse their children are correctly characterized by all of the following
statements,EXCEPT:
A. they are more likely to be alcoholics than are non-abusive parents
B. they are found in all social classes
63
C. they are psychologically immature
D. they are more likely to be men than women

64

One of the following is NOT descriptive of a young population:


A. population pyramid with a wide base
B. median age of 20-24 years old
C. crude birth rate of greater than 30 per 1,000
D. dependency ratio 1:1

Because of his inherent weakness, this method is not utilized in the collection of scientific data:
A. census
B. questionnaire method
65
C. registration method
D. case record method

Niyog-niyogan is a plant whose seeds are used for the treatment of:
A. diarrhea
B. ascariasis
66
C. intestinal colic
D. fever
Which of the following pattern of disease occurrence does not reflect change with the passage of
time:
A. epidemic occurrence
B. endemic occurrence
67
C. secular trends
D. cystic fluctuations

The patient with mumps usually presents with initial complaints of:
A. painful salivary glands
B. otalgia
68
C. enlarged salivary glands
D. deafness

When is preventive medicine considered effective:


A. the setting of a disease is inhibited
B. upon immediate recognition of the manifestations
69
C. prevents the progress of the disease
D. when there is rapid recovery of patients

The three levels of medical care are linked through the :


A. Philippine Health Care System
B. coordinating system
70
C. referral system
D. direct entry point, independent of levels

A disease usually occurring at an endemic level may occur in epidemic proportions when:
A. there is an increased opportunity for transmission
B. its agent undergoes mutation resulting in increased virulence
71
C. a new strain of its agent is introduced
D. there is significant reduction in the herd susceptibility

The primary concern of Environmental Sanitation is:


A. Health education
B. Treatment of disease caused by the environment
72
C. Promotion of health
D. Prevention of disease

The most common site for cancer in persons in developing countries is:
A. cervix and uterus
B. breast
73
C. lung
D. pharynx and oral cavity

If 300 mothers die during the time when there is a total of 1,000,000 pregnancies,the maternal
mortality rate is :
A. 3/1000
74 B. 30/1000
C. 0.3/1000
D. 0.1%
Which of the following statements describing the risk of death from simultaneous smoking and the
use of oral contraceptives is true:
A. A 39-year old non-smoker taking oral contraceptives is at greater risk than a 30-years old
smoker taking the same contraceptives.
75 B. Smoking increases the risk of death of all women of any age who are using contraceptives
C. Smoking does not increase the risk of death in a women taking oral contraceptives unless than
years of age
D. Women under age 20 years and those over age 40 years who smoke and take oral
contraceptives are the two groups at the greatest risk

What statistical rate measures the development of a disease in a community exposed to the risk of
the disease in a period of time:
A. Case fatality
76 B. Maternal mortality
C. Cause of death
D. Incidence

The type of epidemic where majority of the cases develop in one incubation period is called:
A. explosive epidemic
B. insidious epidemic
77
C. progressive epidemic
D. none of the above

The following is NOT a side effect of DPT immunization:


A. abscess
B. seizures
78
C. lymphadenopathy
D. fever

Purification by rapid sand filtration is a :


A. Biological method
B. Chemical method
79
C. Mechanical method
D. None of the above

The main target group of the Philippine Nutrition Program are the following,EXCEPT:
A. elderly people
B. pre-school children
80
C. pregnant and lactating mothers
D. infants

What kind of hypothesis assumes that an association exists:


A. one-tailed
B. alternative
81
C. two-tailed
D. null

This part of research paper provides the background of the study:


A. introduction
B. abstract
82
C. review of related literature
D. statement of the problem
83

The following are characteristics of a classical type of an epidemic curve, EXCEPT:


A. A longer descending limb
B. More deaths in the ascending limb
C. A short ascending limb
D. Slow transmission of the disease
This specifically consist of a core list of drugs approved and authorized by the Department of
Health:
A. National Drug Policy
B. Essential Drug List
84
C. Generic Law
D. National Drug Formulary

The most preferred method of garbage disposal in the Philippines is :


A. burying
B. zymothermic process
85
C. anaerobic decomposition
D. aerobic decomposition

The following are the major skills required to be an effective manager in an organization,EXCEPT:
A. supervisory
B. technical
86
C. conceptual
D. human

A 40-year old physician, a known hypertensive, quadriplegic due to repeated strokes, financially
stable, requested that no resuscitations be performed if he goes into cardiac arrest. Which is the
best consideration to take:
A. Request should be denied
87 B. Think of the consequences as to procedure and result to the patient, family,the health service
and society
C. Get the family's consent
D. Follow the patient's wishes

Motor vehicles emit gas pollutants which is :


A. sulfur
B. nitrogen
88
C. lead
D. hydrocarbon

In the Philippine Health Picture,this is shown as the leading cause of mortality:


A. vehicular accidents
B. cardiovascular diseases
89
C. malnutrition
D. communicable diseases
Which of the following is NOT indicative of the existence of overpopulation in the Philippines:
A. increasing number of unemployed
B. pneumonia continued to be leading cause of death
90
C. increase in squatters area
D. literacy rate of 89%
Which measure/s is/are the most effective and practical in controlling tuberculosis in the
Philippines:
A. mass chemoprophylaxis with INH
B. identification and treatment of infected patients
91
C. mass chest x-rays
D. mass BCG vaccination

Prevention of neonatal tetanus includes the following measures,EXCEPT:


A. community immunization programs for women of reproductive age
B. immunize newborn at birth
92
C. training midwives and traditional birth attendants
D. maternal immunization

The most prevalent mental health disorder in young children is:


A. autism
B. mental retardation
93
C. behavioral problems
D. depression

Health care delivery system:


A. brings health care closer to the people
B. maximized utilization of resources trough integration of health facilities
94
C. implementing of community linkages through the allocation of health centers to barangay
D. achieves all of the above

Among the following,this has the least influence on an individual's resistance to disease:
A. race
B. age
95
C. sex
D. marital status

Among the entries for causes of death in the death certificate,the most useful data which will help
prevent deaths by cutting the chain of events is the :
A. immediate cause
96 B. antecedent cause
C. underlying cause
D. other causes contributing to death

The following statements suggest a common vehicle transmission,EXCEPT:


A. the majority of the cases developed within one incubation period
B. majority of the cases were exposed to the disease agent simultaneously
97
C. majority of the cases cannot be traced to an existing case
D. majority of the cases developed after a short generation time

What is the best index utilized when making field studies on energy nutrient malnutrition:
A. dietary questionnaires
B. physiological abnormalities
98
C. anthropometrics measurement
D. biochemical test
A given set of observations are 3,6,7,8,9,10. What is the median of this given set of observations:
A. 7
B. 8.5
99
C. 7.5
D. 8

Basic health services are provided to the community by:


A. Central level
B. Intermediate level
100
C. Local level
D. All of the above
D

Rehabilitation of the environment means non-chemical means of control like sanitation,


D mechanical blocks & sealing/exclusion of holes
C

D measures of central tendency - mean, median, mode

BONUS
B

D
B A ang sagot ni maam, but this is B. cross-sectional study is a PREVALENCE STUDY

D
BONUS C kay maam but because of devolution, D na dapat

C Age and sex distribution are just secondary to the primary concern which is population enumeration.—Mendoza et

number of deaths among age>50 x 100/ number of deaths in same year


C
A

D D din sa exam ni dr.sigua

methods of hypothesis testing regarding etiology of disease:


1. Method of agreement-A single factor is common in different circumstances in which a
disease occurs with high frequency

2. Method of difference-If the frequency of disease is different, in two places or two groups,
the disease may be caused by some particular factor that differs between them.
3. Method of concomitant variation-Frequency of a factor varies in proportion to the
frequency of disease
4. Method of analogy- the distribution of a disease may be sufficiently similar to that of
some other disease that has been more completely investigated
C

A D kay maam but actually pwede. cholesterol stones - OCP use >5 years, pregnancy, hyperchlesterolemia, obesity,

D
B

Associate mercury poisoning (choice D) with (depending upon the route of exposure) GI
disturbance, pneumonitis, renal failure, and CNS involvement. Sulfur dioxide - due to
combustion of fossil fuels. Forms sulfurous acid on contact with moist mucous membranes
- conj and bronchial irritation, epistaxis, delayed pulmo edeme. no neuropathy
D

BONUS - AD kay maam. Attack rate= number of diseased patients from those exposed so 5/20

C
C

C Data collection as part of conduct of research does not having anything to do with acceptability of method or proce

BONUS - CD kay maam

A is definitely correct. A young population has a population pyramid with a wide base.

Median age: Age at which exactly half the population is older and half is younger.
􀂊May be used to describe a population as “young”or “old”. If median age is:
–Less than 20= “young”
–30 or over = “old”
–between 20 to 29= “intermediate age”

( Source: Shryockand Siegel,1976).

Dependency ratio- ratio is the ratio of dependent population to economically productive.


Thus, 1:1 ratio is likely. Dependent is below 15 and beyond 65 and economically
productive is between 15 & 65. Thus, dependency ratio will not approximate 1:1.

CBR is not ideally used to describe any particular age-related population because it
applies to all ages. Its denominator uses the entire population.
BONUS - B

C Registration data( e.g. voters, drivers, etc) are subject to inaccuracy of certain details about persons. Disease reg

B herba buena - fever, niyog-niyogan - ascaris, bayabas - antibacterial, bawang - hpn, -ampalaya - diabetes, sambon
B

BONUS A/C

C
A contraindicated in those >35 and smoking, 180/110 BP, >20 years diabetic,

B NULL: no association. E.g. Failing the board exams is not associated with long-term career satisfaction or lower ea

A
Classical epidemic curve is usually found with water-borne epidemics, characterized by
gradual rise and gradual decline. E.g. typhoid fever. There is relatively slower transmission
of the disease because of the oro-fecal route. The ascending limb is shorter because of
rapid rise of cases but the descending limb becomes longer because later there is
decline in the susceptibles as they develop resistance. In epidemic curves, there is no
way to speak about deaths, as it is a form of histogram where the incubation periods or
the time of onset of illness (x-axis) are plotted vs. the number of cases identified (y-axis).
.
Other types:

Point – one common vehicle; e.g. food poisoning


Bell-shaped- contact-transmitted; e.g. measles
Inverted/u-shaped- vector-borne; e.g. malaria
B

B
D

C depression in well-developed countries

C
C

C
meration.—Mendoza et al
chlesterolemia, obesity, age, native americans. Pigment stones - hemolytic syndromes, biliary tract infection, ileal diseases
bility of method or procedure.Methodology, yes.

ut persons. Disease registers however are accurate data.

laya - diabetes, sambong - diuretic, lagundi - cough, tsaang gubat - diarrhea, akapulko - skin diseases, ulasimang bato - gout
r satisfaction or lower earning capacity
eal diseases
ang bato - gout

You might also like